Questions and Answers in Pain Medicine: A Guide to Board Exams [1 ed.] 303068203X, 9783030682033, 9783030682040

Features 1000 questions written in a format similar to that of ACGME written board exam in Pain Management Answers are p

237 24 5MB

English Pages [299] Year 2021

Report DMCA / Copyright

DOWNLOAD PDF FILE

Table of contents :
Preface
Contents
Part I: Pain Basics
1: Taxonomy of Pain Terms and Syndromes
Questions
Answers
References
2: Anatomy
Questions
Answers
References
3: Neurochemistry and Pain Processing
Questions
Answers
References
4: Pain Signaling Processes
Questions
Answers
References
5: Physical Examination and Assessment
Questions
Answers
References
Part II: Psychology
6: Psychological Evaluation
Questions
Answers
References
7: Psychological Interventions and Pharmacology
Questions
Answers
References
8: Neurophysiologic Testing
Questions
Answers
References
Part III: Common Pain Syndromes
9: Fibromyalgia and Myofascial Pain
Questions
Answers
References
10: Central Pain
Questions
Answers
References
11: Headache Disorders
Questions
Answers
References
12: Post Dural Puncture Headache and Orofacial Pain
Questions
Answers
References
13: Neck Pain and Cervicogenic Headaches
Questions
Answers
References
14: Back Pain
Questions
Answers
References
15: Pelvic Pain
Questions
Answers
References
16: Painful Neuropathies
Questions
Answers
References
17: Complex Regional Pain Syndrome and Post-herpetic Neuralgia
Questions
Answers
References
18: Post-amputation Pain
Questions
Answers
References
19: Pain in the Emergency Department and Sickle Cell Pain
Questions
Answers
References
20: Cancer Pain and End of Life Care
Questions
Answers
References
21: Critical Illness and Geriatrics
Questions
Answers
References
22: Pediatrics, Pregnancy, and Lactation
Questions
Answers
References
Part IV: Medications
23: Neuropathic Pain Medications
Questions
Answers
References
24: Non-opioid Analgesics
Questions
Answers
References
25: Opioids
Questions
Answers
References
26: Urine Drug Testing and Substance Use Disorders
Questions
Answers
References
27: Intravenous Infusions
Questions
Answers
References
28: Intrathecal Medications and Anesthetic Blockades
Questions
Answers
References
29: Cannabinoids and Herbal Medicine
Questions
Answers
References
Part V: Interventional Topics
30: Fluoroscopy and Radiation Safety
Questions
Answers
References
31: Head/Neck and Truncal Blocks
Questions
Answers
References
32: Visceral Sympathetic Blocks
Questions
Answers
References
33: Radiofrequency, Discography, and Vertebral Augmentation
Questions
Answers
References
34: Neurostimulation
Questions
Answers
References
Part VI: Other Topics
35: Ablative Neurosurgical Procedures
Questions
Answers
References
36: Electrodiagnostics
Questions
Answers
References
37: Rehabilitation Approach
Questions
Answers
References
Recommend Papers

Questions and Answers in Pain Medicine: A Guide to Board Exams [1 ed.]
 303068203X, 9783030682033, 9783030682040

  • 0 0 0
  • Like this paper and download? You can publish your own PDF file online for free in a few minutes! Sign Up
File loading please wait...
Citation preview

Questions and Answers in Pain Medicine A Guide to Board Exams Michael Suer Nalini Sehgal

123

Questions and Answers in Pain Medicine

Michael Suer • Nalini Sehgal

Questions and Answers in Pain Medicine A Guide to Board Exams

Michael Suer University of Wisconsin–Madison Madison, WI USA

Nalini Sehgal University of Wisconsin–Madison Madison, WI USA

ISBN 978-3-030-68203-3    ISBN 978-3-030-68204-0 (eBook) https://doi.org/10.1007/978-3-030-68204-0 © The Editor(s) (if applicable) and The Author(s), under exclusive license to Springer Nature Switzerland AG 2021 This work is subject to copyright. All rights are solely and exclusively licensed by the Publisher, whether the whole or part of the material is concerned, specifically the rights of translation, reprinting, reuse of illustrations, recitation, broadcasting, reproduction on microfilms or in any other physical way, and transmission or information storage and retrieval, electronic adaptation, computer software, or by similar or dissimilar methodology now known or hereafter developed. The use of general descriptive names, registered names, trademarks, service marks, etc. in this publication does not imply, even in the absence of a specific statement, that such names are exempt from the relevant protective laws and regulations and therefore free for general use. The publisher, the authors, and the editors are safe to assume that the advice and information in this book are believed to be true and accurate at the date of publication. Neither the publisher nor the authors or the editors give a warranty, expressed or implied, with respect to the material contained herein or for any errors or omissions that may have been made. The publisher remains neutral with regard to jurisdictional claims in published maps and institutional affiliations. This Springer imprint is published by the registered company Springer Nature Switzerland AG The registered company address is: Gewerbestrasse 11, 6330 Cham, Switzerland

Preface

Studying for board licensing examinations within the practice of medicine is an undertaking of which simply the mention can intensify stress in even the best of students. In my own journey from medical school through physiatry residency and pain fellowship at the University of Wisconsin—Madison, I found book-study to be an arduous but necessary task. To best augment studying, many learners have found that answering as many questions as possible helps gauge current knowledge and hone in on weaknesses, providing guidance for future hours of study. When tasked with leading the board-study didactic schedule for our pain medicine fellowship, my aim was to truly test our fellows’ knowledge on a regular basis through questions that represent what they were likely to see on their upcoming board examinations and were pertinent to their future practices in pain management. Rather than a primary study tool, this work should primarily be used as an enhancement to test one’s current knowledge and come to a better appreciation of one’s own knowledge gaps. We have added explanations to each answer with citations for your further reference. Beyond explaining reasoning of the respective answer being correct, we have also attempted to explain the errors with incorrect answers. It is within these details that true knowledge can be gained as understanding incorrect choices (or not “as correct” in some cases) requires extensive knowledge beyond simply knowing the correct answer. With that note, if there be disagreement with answers, I welcome further discussion to increase my own knowledge. Madison, WI, USA Madison, WI, USA 

Michael Suer Nalini Sehgal

v

Contents

Part I Pain Basics 1 Taxonomy of Pain Terms and Syndromes�����������������������������������    3 Questions������������������������������������������������������������������������������������������    3 Answers��������������������������������������������������������������������������������������������    5 References����������������������������������������������������������������������������������������    8 2 Anatomy������������������������������������������������������������������������������������������    9 Questions������������������������������������������������������������������������������������������    9 Answers��������������������������������������������������������������������������������������������   11 References����������������������������������������������������������������������������������������   14 3 Neurochemistry and Pain Processing������������������������������������������   15 Questions������������������������������������������������������������������������������������������   15 Answers��������������������������������������������������������������������������������������������   17 References����������������������������������������������������������������������������������������   18 4 Pain Signaling Processes����������������������������������������������������������������   19 Questions������������������������������������������������������������������������������������������   19 Answers��������������������������������������������������������������������������������������������   22 References����������������������������������������������������������������������������������������   25 5 Physical Examination and Assessment����������������������������������������   27 Questions������������������������������������������������������������������������������������������   27 Answers��������������������������������������������������������������������������������������������   29 References����������������������������������������������������������������������������������������   31 Part II Psychology 6 Psychological Evaluation��������������������������������������������������������������   35 Questions������������������������������������������������������������������������������������������   35 Answers��������������������������������������������������������������������������������������������   39 References����������������������������������������������������������������������������������������   43 7 Psychological Interventions and Pharmacology�������������������������   45 Questions������������������������������������������������������������������������������������������   45 Answers��������������������������������������������������������������������������������������������   48 References����������������������������������������������������������������������������������������   52

vii

viii

8 Neurophysiologic Testing��������������������������������������������������������������   55 Questions������������������������������������������������������������������������������������������   55 Answers��������������������������������������������������������������������������������������������   56 References����������������������������������������������������������������������������������������   58 Part III Common Pain Syndromes 9 Fibromyalgia and Myofascial Pain����������������������������������������������   61 Questions������������������������������������������������������������������������������������������   61 Answers��������������������������������������������������������������������������������������������   63 References����������������������������������������������������������������������������������������   67 10 Central Pain������������������������������������������������������������������������������������   69 Questions������������������������������������������������������������������������������������������   69 Answers��������������������������������������������������������������������������������������������   71 References����������������������������������������������������������������������������������������   74 11 Headache Disorders ����������������������������������������������������������������������   77 Questions������������������������������������������������������������������������������������������   77 Answers��������������������������������������������������������������������������������������������   80 References����������������������������������������������������������������������������������������   85 12 Post Dural Puncture Headache and Orofacial Pain ������������������   87 Questions������������������������������������������������������������������������������������������   87 Answers��������������������������������������������������������������������������������������������   90 References����������������������������������������������������������������������������������������   94 13 Neck Pain and Cervicogenic Headaches��������������������������������������   97 Questions������������������������������������������������������������������������������������������   97 Answers��������������������������������������������������������������������������������������������   98 References����������������������������������������������������������������������������������������   99 14 Back Pain����������������������������������������������������������������������������������������  101 Questions������������������������������������������������������������������������������������������  101 Answers��������������������������������������������������������������������������������������������  106 References����������������������������������������������������������������������������������������  113 15 Pelvic Pain��������������������������������������������������������������������������������������  117 Questions������������������������������������������������������������������������������������������  117 Answers��������������������������������������������������������������������������������������������  119 References����������������������������������������������������������������������������������������  121 16 Painful Neuropathies ��������������������������������������������������������������������  123 Questions������������������������������������������������������������������������������������������  123 Answers��������������������������������������������������������������������������������������������  127 References����������������������������������������������������������������������������������������  131 17 Complex Regional Pain Syndrome and Post-herpetic Neuralgia����������������������������������������������������������������������������������������  133 Questions������������������������������������������������������������������������������������������  133 Answers��������������������������������������������������������������������������������������������  136 References����������������������������������������������������������������������������������������  140

Contents

Contents

ix

18 Post-amputation Pain��������������������������������������������������������������������  143 Questions������������������������������������������������������������������������������������������  143 Answers��������������������������������������������������������������������������������������������  144 References����������������������������������������������������������������������������������������  146 19 Pain in the Emergency Department and Sickle Cell Pain����������  149 Questions������������������������������������������������������������������������������������������  149 Answers��������������������������������������������������������������������������������������������  151 References����������������������������������������������������������������������������������������  154 20 Cancer Pain and End of Life Care ����������������������������������������������  155 Questions������������������������������������������������������������������������������������������  155 Answers��������������������������������������������������������������������������������������������  158 References����������������������������������������������������������������������������������������  162 21 Critical Illness and Geriatrics������������������������������������������������������  165 Questions������������������������������������������������������������������������������������������  165 Answers��������������������������������������������������������������������������������������������  170 References����������������������������������������������������������������������������������������  176 22 Pediatrics, Pregnancy, and Lactation������������������������������������������  179 Questions������������������������������������������������������������������������������������������  179 Answers��������������������������������������������������������������������������������������������  184 References����������������������������������������������������������������������������������������  192 Part IV Medications 23 Neuropathic Pain Medications������������������������������������������������������  197 Questions������������������������������������������������������������������������������������������  197 Answers��������������������������������������������������������������������������������������������  201 References����������������������������������������������������������������������������������������  207 24 Non-opioid Analgesics��������������������������������������������������������������������  209 Questions������������������������������������������������������������������������������������������  209 Answers��������������������������������������������������������������������������������������������  211 References����������������������������������������������������������������������������������������  214 25 Opioids��������������������������������������������������������������������������������������������  217 Questions������������������������������������������������������������������������������������������  217 Answers��������������������������������������������������������������������������������������������  220 References����������������������������������������������������������������������������������������  225 26 Urine Drug Testing and Substance Use Disorders����������������������  227 Questions������������������������������������������������������������������������������������������  227 Answers��������������������������������������������������������������������������������������������  231 References����������������������������������������������������������������������������������������  235 27 Intravenous Infusions��������������������������������������������������������������������  237 Questions������������������������������������������������������������������������������������������  237 Answers��������������������������������������������������������������������������������������������  238 References����������������������������������������������������������������������������������������  239

x

28 Intrathecal Medications and Anesthetic Blockades��������������������  241 Questions������������������������������������������������������������������������������������������  241 Answers��������������������������������������������������������������������������������������������  244 References����������������������������������������������������������������������������������������  248 29 Cannabinoids and Herbal Medicine��������������������������������������������  249 Questions������������������������������������������������������������������������������������������  249 Answers��������������������������������������������������������������������������������������������  251 References����������������������������������������������������������������������������������������  253 Part V Interventional Topics 30 Fluoroscopy and Radiation Safety ����������������������������������������������  257 Questions������������������������������������������������������������������������������������������  257 Answers��������������������������������������������������������������������������������������������  259 References����������������������������������������������������������������������������������������  262 31 Head/Neck and Truncal Blocks����������������������������������������������������  263 Questions������������������������������������������������������������������������������������������  263 Answers��������������������������������������������������������������������������������������������  266 References����������������������������������������������������������������������������������������  270 32 Visceral Sympathetic Blocks ��������������������������������������������������������  273 Questions������������������������������������������������������������������������������������������  273 Answers��������������������������������������������������������������������������������������������  275 References����������������������������������������������������������������������������������������  278 33 Radiofrequency, Discography, and Vertebral Augmentation������  279 Questions������������������������������������������������������������������������������������������  279 Answers��������������������������������������������������������������������������������������������  281 References����������������������������������������������������������������������������������������  284 34 Neurostimulation����������������������������������������������������������������������������  287 Questions������������������������������������������������������������������������������������������  287 Answers��������������������������������������������������������������������������������������������  289 References����������������������������������������������������������������������������������������  291 Part VI Other Topics 35 Ablative Neurosurgical Procedures����������������������������������������������  295 Questions������������������������������������������������������������������������������������������  295 Answers��������������������������������������������������������������������������������������������  296 References����������������������������������������������������������������������������������������  297 36 Electrodiagnostics��������������������������������������������������������������������������  299 Questions������������������������������������������������������������������������������������������  299 Answers��������������������������������������������������������������������������������������������  300 References����������������������������������������������������������������������������������������  301 37 Rehabilitation Approach ��������������������������������������������������������������  303 Questions������������������������������������������������������������������������������������������  303 Answers��������������������������������������������������������������������������������������������  306 References����������������������������������������������������������������������������������������  310

Contents

Part I Pain Basics

1

Taxonomy of Pain Terms and Syndromes

Questions 1. 65-year-old patient who suffered radius and ulnar fracture 6  months ago has continuing pain in her arm. She also experiences pain to light touch including breezes, color changes, edema, and has hair loss. Pain to light touch is best defined as A. Chronic pain B. Hyperalgesia C. Allodynia D. Dysesthesia 2. Which of the following is not used to diagnose CRPS according to the Budapest Criteria A. Increased uptake on triple phase bone scan B. Evidence of hyperalgesia to pinprick and/or allodynia C. Edema and/or sweating changes and/or sweating asymmetry D. No other diagnosis that better explains the signs and symptoms 3. Patient has a minor MVC with whiplash noted by the patient. ED workup was negative for fractures and she was discharged. She presents to your clinic 3  months later with continued pain noted to be electric, sharp, stabbing pain in the jaw area. In-office x-rays of the neck are shown. What radiologic abnormality is seen in Fig. 1.1? A. Cervical osteoarthritis B. Eagle Syndrome

Fig. 1.1

C. Glossopharyngeal neuralgia D. Myofascial pain 4. Cubital tunnel syndrome is entrapment of the ulnar nerve between the following A. Trochlear groove—olecranon process B. Ligament of Struthers—humerus C. Palmar carpal ligament—hypothenar muscles D. Arcade of Frohse

© The Author(s), under exclusive license to Springer Nature Switzerland AG 2021 M. Suer, N. Sehgal, Questions and Answers in Pain Medicine, https://doi.org/10.1007/978-3-030-68204-0_1

3

4

5. Loss of ability to perform a specific task in a standard or normal fashion A. Disability B. Handicap C. Physical impairment 6. In testing lower extremity dermatomes, a patient notes he can feel you touching the lateral aspect of his leg, “but it doesn’t feel the same” though it is not sensed as painful or unpleasant. This is best described as A. Paresthesia B. Dysesthesia C. Anesthesia D. Anesthesia dolorosa 7. In testing of lower extremity dermatomes, a patient can feel you touching the lateral aspect of his leg and notes it to be quite unpleasant A. Paresthesia B. Dysesthesia C. Anesthesia D. Anesthesia dolorosa 8. Patient presents with pain diagram full of aching pain with a frowning face drawn on the pictogram as well. She notes that she has had full body pain the last several months and feels that it is impacting every aspect of her life. She is very disturbed by these feelings and note that it is “taking over my life.” On palpation, she has multiple tender spots noted in the upper back, but none elsewhere. You perform the Widespread Pain Index and she scores 4. She then scores 10. What is the diagnosis A. Myofascial pain B. Pain Catastrophizing C. Fibromyalgia D. Malingering 9. Increased pain to a stimulus that normally provokes pain A. Hyperalgesia B. Allodynia C. Dysesthesia D. Hyperesthesia 10. 78-year-old woman presents to the clinic with pain over the last 6 months, when she suffered a left-MCA stroke. Functionally,

1  Taxonomy of Pain Terms and Syndromes

she has mild weakness still noted in the right upper extremity. Her pain is described as burning in the right arm. This is best described as A. Somatic pain B. Central neuropathic pain C. Peripheral neuropathic pain D. Neuropathy 11. 34-year-old woman presents with vague abdominal pain. She describes it described as sickening, deep, squeezing, and dull. GI and OB work-up revealed a uterine fibroid. Her pain is throughout her lower abdomen without localization. This is best described as A. Somatic pain B. Visceral pain C. Malingering D. Nociceptive pain 12. Pain is an unpleasant _________ and __________ associated with actual or potential tissue damage, or described in terms of damage A. Nociceptive, noxious B. Sensory, emotional C. Sensory, noxious D. Nociceptive, emotional 13. 34-year-old man with traumatic amputation of his left arm at the proximal humerus. He initially notes pain in the missing limb as well as a feeling of the hand still being present. Over time, the pain goes away, but he has noted the hand to still feel as though it is there. However, the hand feels closer to the residual limb than previously noted as though his hand is attached to the humerus. This is best described as A. Phantom pain B. Telescoping C. Neuropathic pain D. Alien hand syndrome 14. 21-year-old rehab resident is golfing and feels sudden, sharp pain in his back radiating down the posterior aspect of his right leg. He initially takes a Medrol Dosepak, but does not improve. He undergoes physical therapy for 3  months without improvement and presents to your clinic. On exam, he has ­ decreased sensation in the posterior aspect of

Answers

his leg. Strength and reflexes are normal. This is best described as A. Radicular pain B. Radiculopathy C. Peripheral neuropathy D. Discogenic pain Use the following question stem to answer questions 15 through 17. 79-year-old patient with history of diabetes, heart disease, and chronic back pain due to central canal stenosis presents with gradual onset lower limb symptoms. 15. He described burning pain noted in both feet that is always present. There is no change with activity or positioning A. Peripheral neuropathy B. Radiculopathy C. Neurogenic claudication D. Vascular claudication 16. He describes burning pain that is improved with walking uphill and leaning forward on his walker. He is able to walk long distances with these postural changes A. Peripheral neuropathy B. Radiculopathy C. Neurogenic claudication D. Vascular claudication 17. He describes burning pain that is present with ambulation of 400  ft no matter of positioning A. Peripheral neuropathy B. Radiculopathy C. Neurogenic claudication D. Vascular claudication 18. Which of the following referred pain patterns is not consistent with those classically described A. Shoulders—lung and diaphragm B. Right tip of scapula—gallbladder C. Left shoulder—spleen D. Left tip of scapula—liver 19. Tarsal tunnel contains all of the following except: A. Posterior tibial artery B. Flexor digitorum longus tendon C. Tibial nerve D. Extensor hallucis longus tendon

5

20. Patient presents with pain in the neck radiating down the right arm into the hand. You have the patient seated with their arm abducted 30° at the shoulder and maximally extended. The radial pulse is palpated at the wrist. The patient then extends their neck and turns the head toward the ipsilateral shoulder, takes a deep breath, and holds it. You notice the pulse is diminished. This test is described as: A. Eden’s test B. Adson’s test C. Roo’s test D. Tom Foolery’s test 21. Common causes of thoracic outlet syndrome include the following except: A. Hypertrophic scalene B. Cervical rib C. Post-traumatic fibrosis D. Hypertrophied pectoralis muscle E. None of the above 22. The percentage of pain relief that is the commonly accepted standard considered as a satisfactory result is: A. 30% B. 50% C. 70% D. 80%

Answers 1. C [1] A. Chronic pain—Pain that persists beyond the course of an acute disease or a reasonable time for an injury to heal or that is associated with a chronic pathologic process that causes continuous pain or the pain recurs at intervals of months or years. Some investigators use duration of ≥6 months to designate pain as chronic. B. Hyperalgesia—Increased pain from a stimulus that normally provokes pain. C. Allodynia—Pain due to a stimulus that does not normally provoke pain. D. Dysesthesia—An unpleasant abnormal evoked sensation, whether spontaneous or evoked.

6

2. A [1, 2] CRPS is a painful condition characterized by a continuing regional pain that is seemingly disproportionate in time or degree to the usual course of any known trauma or other lesion. The pain is regional (not in a specific nerve territory or dermatome) and usually has a distal predominance of abnormal sensory, motor, sudomotor, vasomotor, and/or trophic findings. The syndrome shows variable progression over time. While a triple phase bone scan can be used to help confirm the diagnosis of CRPS, it is not included in the Budapest criteria. 3. B [1] Elongated or deviated styloid process and/or calcification of the stylohyoid ligament, which interferes with adjacent anatomical structures giving rise to pain. Pain may be precipitated by trauma to the region. 4. A [1, 3, 4] Cubital tunnel syndrome is entrapment of the ulnar nerve in a fibro-osseous tunnel formed by the trochlear groove between the olecranon process and the medial epicondyle of the humerus. Entrapment at the arcade or ligament of Struthers is proximal to this location. The ulnar nerve can be trapped at the wrist in multiple locations including at the palmar carpal ligament. The arcade of Frohse, a common site of radial nerve entrapment, is the most superior part of the superficial layer of the supinator muscle, and is a fibrous arch over the posterior interosseous nerve. 5. A [1, 5] A. Disability: A restriction or inability to perform an activity in the manner or within the range considered normal for a human being, mostly resulting from impairment. B. Handicap: This is the result of an impairment or disability that limits or prevents the fulfilment of one or several roles regarded as normal, depending on age, sex and social and cultural factors. C. Impairment: Any temporary or permanent loss or abnormality of a body struc-

1  Taxonomy of Pain Terms and Syndromes

ture or function, whether physiological or psychological. An impairment is a disturbance affecting functions that are essentially mental (memory, consciousness) or sensory, internal organs (heart, kidney), the head, the trunk or the limbs. 6. A [1] A. Paresthesia: An abnormal sensation (not unpleasant), whether spontaneous or evoked. B. Dysesthesia: An unpleasant abnormal evoked sensation, whether spontaneous or evoked. C. Anesthesia: Absence of all sensory modalities. D. Anesthesia dolorosa: Pain in an area or region that is anesthetic. 7. B [1] A. Paresthesia: An abnormal sensation (not unpleasant), whether spontaneous or evoked. B. Dysesthesia: An unpleasant abnormal evoked sensation, whether spontaneous or evoked. C. Anesthesia: Absence of all sensory modalities. D. Anesthesia dolorosa: Pain in an area or region that is anesthetic. 8. C [1] While the use of widespread pain with tenderness on palpation of 11 of 18 locations, these criteria are no longer used to diagnose fibromyalgia. To diagnosis fibromyalgia, a score of 7 or greater in the widespread pain index (WPI) and symptom severity (SS) score of 5 or greater (or WPI 3–6 and SS score ≥9) are required. While this individual may have myofascial pain, the WPI and SS scores make fibromyalgia more likely. 9. A [1] A. Hyperalgesia: Increased pain from a stimulus that normally provokes pain. B. Allodynia: Pain due to a stimulus that does not normally provoke pain. C. Dysesthesia: An unpleasant abnormal evoked sensation, whether spontaneous or evoked.

Answers

10.

11.

12.

13.

D. Hyperesthesia: Increased sensitivity to stimulation; this excludes the special senses. B [1] Pain caused by a lesion or disease of the central somatosensory nervous system. Central pain is usually associated with abnormal sensibility to temperature and to noxious stimulation. Given the onset of pain with her stroke occurrence, this is likely post-stroke pain due to a central lesion. B [1] A. Somatic pain: Pain carried along the sensory fibers; this pain is usually discrete and intense. B. Visceral pain: pain that results from the activation of nociceptors of the thoracic, pelvic, or abdominal viscera (organs) C. Malingering: to exaggerate or feign illness in order to escape duty or work. There is nothing in the stem to suggest she is trying to escape duty or work. D. Nociceptive pain: Pain caused by activation of nociceptive afferent fibers. This type of pain satisfies the criteria for pain transmission (i.e., transmission to the spinal cord, thalamus, and then to the cerebral cortex). B [1] An unpleasant sensory and emotional experience associated with actual or potential tissue damage, or described in terms of damage. Individuals with pain asymbolia experience pain without the emotional unpleasantness that accompanies it. B [1] A. Phantom pain: Pain referred to a surgically removed limb or portion thereof. B. Telescoping: The phantom sensation gradually approaches the residual limb and eventually becomes located inside the residual limb. C. Neuropathic pain: Pain initiated or caused by a primary lesion or disease in the peripheral or central somatosensory nervous systems. D. Alien hand syndrome: category of conditions in which a person experiences their

7

limbs acting seemingly on their own, without conscious control over the actions. 14. B [1] A. Radicular pain: Pain perceived as arising in a limb or the trunk wall caused by ectopic activation of nociceptive afferent fibers in a spinal nerve or its roots or other neuropathic mechanisms. B. Radiculopathy: Objective loss of sensory and/or motor function as a result of conduction block in axons of a spinal nerve or its roots. Given the patient’s presentation with loss of sensation, this is the best answer. C. Peripheral neuropathy: Constant or intermittent burning, aching, or lancinating limb pain due to generalized or focal diseases of peripheral nerves D. Discogenic pain: pain originating from a damaged vertebral disc, particularly due to degenerative disc disease or annular tear in the disc. However, not all disc pathology causes pain. 15–17. Use the following descriptions to answer each of the question stems within questions 15–17 [1] A. Peripheral neuropathy: Constant or intermittent burning, aching, or lancinating limb pain due to generalized or focal diseases of peripheral nerves. B. Radiculopathy: Objective loss of sensory and/or motor function as a result of conduction block in axons of a spinal nerve or its roots. Symptoms C. Neurogenic claudication: Crampy, achy lower extremity pain precipitated by ambulation particularly with upright posture and relieved with leaning forward seen in spinal canal stenosis. D. Vascular claudication: Crampy, achy lower extremity pain precipitated by ambulation and activity, caused by vascular insufficiency. This is position independent, i.e. it is not relieved with leaning forward at the waist as neurogenic claudication due to spinal canal

8

stenosis but the individual must stop activities regardless of positioning. They often have a very predictable walking distance to symptom onset. 15. A—Peripheral neuropathy He described burning pain noted in both feet that is always present. There is no change with activity or positioning 16. C—Neurogenic claudication He describes burning pain that is improved with walking uphill and leaning forward on his walker. He is able to walk long distances with these postural changes 17. D—Vascular claudication He describes burning pain that is present with ambulation of 400  ft no matter of positioning 18. D The above associations are correct except the liver which provides referred pain to the lower corner of the right scapulae as does the gall bladder. 19. D [1] The contents of the tarsal tunnel are the tibialis posterior tendon, flexor digitorum longus tendon, posterior tibial artery and vein, tibial nerve, and flexor hallucis longus tendon. Irritation of the posterior tibial nerve as it passes through the fibrous tarsal tunnel in the foot, resulting in foot pain with neuropathic features. 20. B [6] The test is positive if there is a marked decrease, or disappearance, of the radial pulse. It is important to check the patient’s radial pulse on the other arm to recognize the patient’s normal pulse. A. Eden’s test involves asking the client to push out the chest and pull the shoulder girdles back, as if assuming a military position of attention, while the therapist palpates the strength of the radial pulse B. Adson’s test: Correct C. Roo’s test: Have the patient abduct each shoulder to 90° with the shoulders laterally rotated and the elbows flexed slightly behind the frontal plane. Instruct the patient to open and close their hands slowly for 3 min. A positive test is reported if the patient is unable to keep their arms

1  Taxonomy of Pain Terms and Syndromes

in the starting position for 3 min or if ischemic pain, heaviness, or weakness is present in the arm or if the patient reports numbness or tingling in the hand during the test. D. Tom Foolery’s test: The inventor Tom Edison developed his own test to administer to applicants to his employment. It was so difficult, it became known as Tom Foolery’s test. This will not be on your boards 21. E [1] Pain in the root of the neck, head, and shoulder, radiating down the arm into the hand, due to compression of the brachial plexus by a hypertrophied scalene or pectoralis muscle, congenital bands, posttraumatic fibrosis, cervical rib or band, or a malformed first thoracic rib. 22. B [5] The commonly accepted standard for satisfactory self-reported pain relief after an intervention is 50%. This standard, while important, does not apply universally to all patients or in all situations, particularly in cases of chronic noncancer pain, where other outcomes (such as function) may assume greater importance.

References 1. Chekka K, Benzon HT. Taxonomy: definition of pain terms and chronic pain syndromes. In: Benzon HT, editor. Essentials of pain medicine: Elsevier; 2018. 2. Harden RN, Bruehl S, Stanton-Hicks M, Wilson PR.  Proposed new diagnostic criteria for complex regional pain syndrome. Pain Med. 2007;8:326–31. 3. Fuss FK.  The eponym of the supinator arch. Surg Radiol Anat. 1996;18(2):158. 4. Caetano EB, Sabongi Neto JJ, Vieira LA, Caetano MF.  The arcade of Struthers: an anatomical study and clinical implications. Rev Bras Ortop. 2017;52(3):331–6. https://doi.org/10.1016/j.rboe. 2016.07.006. Published 2017 May 9. 5. Rondinelli RD, Ranavaya M.  Disability assessment. In: Benzon HT, et al., editors. Practical management of pain. Philadelphia, PA: Elsevier Mosby; 2014. 6. Kaplan J, Kanwal A.  Thoracic outlet syndrome. [Updated 4 May 2020]. In: StatPearls [Internet]. Treasure Island, FL: StatPearls Publishing; 2020. https://www.ncbi.nlm.nih.gov/books/NBK557450/.

2

Anatomy

Questions 1. Which of the following does not occur with selective injury to the anterior spinal artery? A. Dense motor paralysis B. Variable sensory loss C. Position and vibration sensory loss D. Bowel and bladder incontinence 2. Nerves arising from the lumbar plexus include 1. Femoral 2. Obturator 3. Ilioinguinal 4. Sciatic A. 1, 2, 3 B. 1, 3 C. 2, 4 D. All of the above 3. Which of the following is true of the superior hypogastric plexus? A. Is located at the level of the origin of the renal arteries B. Does not contain parasympathetic fibers C. Gives off branches to the testicular or ovarian plexus D. Is oriented in a transverse plane parallel to the pelvic floor 4. Which of the following is true of the stellate ganglion? A. It lies at the level of the C6–C7 interspace

B. Vertebral artery is anterior to the stellate ganglion C. Pleura is posterior to the stellate ganglion D. Anterior divisions of C8 and T1 are inferior to the stellate ganglion 5. EMG needle evaluation of a patient shows positive waves and long duration motor unit potentials in the biceps and pronator teres. All remaining limb muscles tested are normal. The lesion causing these findings could be in the A. Musculocutaneous nerve, upper trunk, C6 nerve root B. Lateral cord, C6 nerve root, upper trunk C. Median nerve, lateral cord, C7 nerve root D. Anterior division, middle trunk, musculocutaneous nerve 6. Which of the following groups of ribs articulate with one vertebral body only? A. 2, 3, 4, and 5 B. 9, 10, 11, and 12 C. 1, 2, 11, and 12 D. 1, 10, 11, and 12 7. True or False: Both the greater occipital and lesser occipital nerves arise from the dorsal primary rami of the second and third cervical nerves. A. True B. False

© The Author(s), under exclusive license to Springer Nature Switzerland AG 2021 M. Suer, N. Sehgal, Questions and Answers in Pain Medicine, https://doi.org/10.1007/978-3-030-68204-0_2

9

10

8. The greater occipital nerve A. Is lateral to the lesser occipital nerve B. Is lateral to the occipital artery C. Innervates the medial portion of the posterior scalp D. Block infrequently causes local ecchymosis and hematoma 9. Select the correct statement(s) regarding afferent innervation of the female reproductive organs. 1. Afferents from uterus and lower uterine segment arise in T10–T12 and L1 dorsal root ganglion 2. Afferents from uterine cervix accompany sympathetic nerves in the hypogastric plexuses 3. Afferents from vagina travel along the inferior hypogastric plexus and the pudendal nerves 4. Perineum is supplied by the sacral nerves S2 to S4 traveling in the pudendal nerves A. 1 and 3 B. 2 and 4 C. 1, 2 and 3 D. All of the above 10. All of the following are types of forces that occur in the lumbar spine except: A. Torsion B. Shear C. Creep D. Tension 11. The average total range of lumbar flexion-­ extension is A. 30° B. 50° C. 70° D. 90° 12. How many compartments are in the lower leg? A. 2 B. 3 C. 4 D. 5 13. The vertebral bodies are totally dependent on other structures for stability in which plane? A. Vertical B. Coronal

2 Anatomy

C. Horizontal D. Combination of all three 14. The normal lumbar facet joint will accept how many mL of fluid? A. 0.5–1 B. 1.0–1.5 C. 1.5–2 D. 2–2.5 15. How many groups of lumbar muscles are posterior to the transverse processes? A. 2 B. 3 C. 4 D. 5 16. The disc receives direct nutrition via A. Dorsal proximal medial arteries B. Diffusion through the vertebral end plate and vessels in the outer annulus C. Distal radicular arteries D. Direct supply via aortic spinal branches 17. To differentiate between C6 and C7 spinous processes when palpating; A. C6 disappears on neck extension B. C6 disappears on neck flexion C. C6 and C7 remain palpable on neck motion. D. C7 enlarges on neck extension 18. In the mid cervical spine, when the head side bends to the right, the vertebral body rotates A. To the right B. To the left C. Does not rotate D. Alternates right and left rotation 19. Which of the following nerves generally does NOT innervate the perineum? A. Ilioinguinal B. Genitofemoral C. Posterior femoral cutaneous D. Lateral femoral cutaneous 20. The carpal tunnel contains the following structure? A. Transverse carpal ligament B. Radial artery C. Palmaris longus D. Flexor pollicis longus E. Ulnar nerve

Answers

21. The most common direction of shoulder instability is: A. Anterior inferior B. Posterior inferior C. Posterior superior D. Anterior superior E. Medio-lateral 22. You note an area of high intensity in the L4–5 disc of a 28-year-old man with midline low back pain. You suspect discogenic pain. Sensory innervation of the intervertebral discs is supplied by which nerves? A. Gray rami communicantes B. Sinuvertebral nerve C. Posterior longitudinal ligament plexus D. Spinal nerve root E. All of the above

Answers 1. C [1, 2] The anterior spinal artery arises from small branches from the bilateral vertebral arteries. A small midline artery, the anterior spinal artery supplies the ventral two-thirds of the spinal cord. Its continuity is tenuous, and occlusion or interruption of the artery, e.g., from dissection of an aortic aneurysm, can lead to extensive spinal cord infarction. Anterior spinal artery syndrome affects pain and temperature sensation transmitted through the spinothalamic tract in the anterolateral cord, plus motor neurons descending in the ventral horns. Position and vibratory sense are carried in the dorsal columns, which are preserved. 2. A [3] All but the sciatic nerve originate from the lumbar plexus. The sciatic nerve originates from the L4 through S3 roots of the sacral plexus. The roots traverse the anterior surfaces of the lateral sacrum and coalesce on the anterior surface of the piriformis muscle. It provides sensory innervation of the posterior thigh and most of the leg and foot.

11

The femoral nerve is formed by the posterior divisions of L2–4 and descends from the plexus lateral to the psoas muscle. It is responsible for sensory innervation of the anteromedial thigh and the medial leg and foot. The obturator nerve is formed by the anterior divisions of L2–4 and provides sensation to a variable area of the medial thigh. The ilioinguinal nerve is formed from fibers of T12 and L1. It provides sensory innervation to the superomedial thigh, root of the penis and upper part of the scrotum in the male, or mons pubis and labium majus in the female. 3. C [4] The superior hypogastric plexus, also known as the presacral nerve, is a bilateral, retroperitoneal structure positioned in proximity to the common iliac bifurcation, extending from the lower third of the L5 vertebral body to the upper third of the S1 vertebral body at the sacral promontory. It is oriented longitudinally relative to the pelvic floor. Its location permits access for surgical or chemical extirpation. The superior hypogastric plexus contains both sympathetic and parasympathetic fibers. The sympathetic innervation comes from the lumbar sympathetic chains and branches from the aortic plexus, plus fibers traversing the celiac and inferior mesenteric plexuses. Parasympathetic fibers arise in the S2–S4 segments and travel as pelvic splanchnic nerves (nervi erigentes), which pass through the inferior hypogastric plexus to reach the superior hypogastric plexus. The superior plexus supplies branches to the ureteric and testicular/ovarian plexuses, the sigmoid colon and the plexi surrounding the common internal iliac arteries. The superior plexus divides into the left and right hypogastric nerves, which descend lateral to the sigmoid colon and rectosigmoid junction to reach the two inferior hypogastric plexuses.

12

In contrast, the inferior hypogastric plexus is a bilateral structure situated on either side of the rectum, the lower part of the bladder and (in the male) prostate, the seminal vesicles or uterine cervix, and the vaginal fornices. The inferior plexus is horizontally oriented, extending posteroanteriorly and parallel relative to the pelvic floor. Its location and orientation render it poorly accessible to surgical or chemical extirpation. 4. B [5] The stellate ganglion usually lies in front of the neck of the first rib, and extends to the interspace between C7 and T1. It is medially limited by the longus-colli muscle, laterally by the scalene muscles, anteriorly by the subclavian artery, posteriorly by transverse processes and prevertebral fascia, and inferiorly by the posterior aspect of the pleura. At the level of the stellate ganglion the vertebral artery is anterior, having originated from the subclavian artery. After passing over the ganglion the artery enters the vertebral foramen and is located posterior to the anterior tubercle of C6. Other structures posterior to the stellate ganglion are the anterior divisions of the C8 and T1 nerves (inferior aspect of the brachial plexus). 5. B [6] A detailed knowledge of the brachial plexus is crucial to accurate diagnosis and treatment of neuropathic conditions. Answering this question requires tracing the innervation of each muscle from the peripheral nerve back through the plexus to the nerve roots. The biceps is innervated by the musculocutaneous nerve, lateral cord, anterior division, upper trunk, and C5, C6 nerve roots. The innervation of pronator teres is median nerve, lateral cord, anterior division, upper/middle trunks, and C6, C7 nerve roots. Only answer b includes elements common to the innervation of both muscles. 6. D [7, 8] Ribs 1, 10, 11, and 12 are atypical and articulate with the corresponding vertebral body only. The costovertebral joints are the

2 Anatomy

articulations that connect the heads of the ribs with the bodies of the thoracic vertebrae. In all but the “atypical” ribs, two convex facets from the head attach to two adjacent vertebrae. The facet of the tubercle of the rib articulates with the adjacent transverse process of a thoracic vertebra. This is a plane-type synovial joint called the costotransverse joint. Ribs 1–7 articulate anteriorly with the sternum. Ribs 8–10 articulate anteriorly with each other, merging at the costal cartilages. Ribs 11–12 have no anterior articulations (the “floating” ribs). 7. B [9] The LON arises from the ventral primary rami of C2 and C3 spinal nerves, and the GON arises from dorsal primary rami of C2 and C3 spinal nerves. 8. C [9] The GON pierces the fascia just below the superior nuchal ridge, and supplies the medial portion of the posterior scalp as far as the vertex. The occipital artery accompanies and travels lateral to the GON.  The LON passes superiorly along the posterior border of the sternocleidomastoid muscle and supplies the lateral portion of the posterior scalp and cranial surface of the pinna of the ear. GON block is used to diagnose and treat occipital neuralgia. Third occipital nerve (TON) blocks are used to diagnose C2–C3 facet pain. Local ecchymosis/hematoma are not uncommon. 9. D [10] The innervation of the reproductive organs can be divided into five general sections: uterus, lower uterine segment, cervix, vagina and perineum. The uterus and lower uterine segment are innervated by afferents with cell bodies in the dorsal root ganglia of the T10–T12 and L1 segments. Primary afferents from the uterus and cervix travel with the sympathetic nerves in the uterine and cervical plexus, the inferior, middle and superior hypogastric and aortic plexuses to the lumbar sympathetic chain and course cephalad to reach the dorsal root ganglia of T10, T11, T12 and L1 spinal nerves. The

Answers

10.

11. 12.

13.

14. 15.

central fibers of the DRG neurons pass through the dorsal roots of the spinal nerves to synapse with neurons in the dorsal horn. Pain caused by uterine contractions is referred to the T10 to L1 dermatomes. Cervical innervation travels the T10–T12 pathways with the innervation of the uterus and not via the sacral plexus, S2–S4 [11]. The principal fiber types of the uterus and cervix are Aδ (small myelinated) and C (unmyelinated) fibers. The perineum is innervated by S2 to S4 nerve roots via the pudendal nerves. There is some overlap in the vagina, with over 70% of the afferents linking up with the afferents from the uterine-­ cervical area to the inferior hypogastric plexus, while the remainder of the afferents from the vagina and lower genital tract link up to the pudendal nerve. The uterine horns are innervated separately, via the sympathetic nerves [12]. The pudendal nerve and other smaller nerves, derived from S2, S3 and S4 convey pain from the vagina and the perineum in the second and third stages of labor However, the peculiar pain caused by pressure on the intrapelvic structures and felt in the thigh and upper legs usually involves L2 to S3 spinal segments. C [13] While loads on the spine act in combination, these are the major unique forces to be considered. C [14] 70° C [15] There are four lower leg compartments. They are anterior, lateral, superficial posterior, and deep posterior. The tibialis posterior is sometimes classified as its own separate compartment and not grouped with the deep posterior compartment. C [16] There are no hooks, bumps, or ridges on the vertebral bodies that prevent gliding or twisting movements between them. B [17] 1.0–1.5 B [16]

13

16.

17.

18.

19. 20.

21.

(1) The short intersegmental muscles (interspinales, intertransversarii), (2) polysegmental muscles (multifidus, longissimus, iliocostalis), (3) long polysegmental muscles (longissimus, iliocostalis lumborum) B [16] The only vessels that actually enter the discs are small branches from the metaphyseal arteries which anastomose over the outer surface of the annulus fibrosus. Otherwise, discs are dependent upon diffusion for nutrition. A [18] The C6 spinous process disappears from beneath the palpating finger on neck extension, while the spinous process of C7 remains stationary. A [13] In the mid cervical spine (C3–C7) there is a coupling motion whereby sidebending and rotation occur to the same side. In the thoracic and lumbar spine, sidebending to the right results in rotation of the vertebral body to the left. D [19] Lateral femoral cutaneous D [20] The carpal tunnel lies deep to the palmaris longus and is defined by four palpable bony prominences: proximally by the pisiform and tubercle of the scaphoid (navicular) and distally by the hook of hamate and tubercle of the trapezium. The transverse (palmar) carpal ligament is on its volar surface and attaches to these four prominences to form a fibrous sheath which contains the carpal tunnel. The carpal tunnel contains the median nerve, and the finger flexor tendons from the forearm to the hand (four flexor digitorum superficialis tendons, four flexor digitorum profundus tendons and one flexor pollicis longus tendon). It does not contain the radial artery and the lumbrical muscles. A [21, 22] While with trauma, dislocations can occur in almost any direction, the shoulder capsule is weakest anteriorly and instability is therefore most common in this direction. The unstable shoulder may sublux (be partially displaced from normal position) and then reduce sponta-

14

neously, or may dislocate (be fully displaced; does not reduce spontaneously). Multidirectional instability can occur and is more common in patients with lax ligaments (e.g. joint hypermobility syndrome, Ehlers-Danlos syndrome, Marfan syndrome). Consider multidirectional instability in patients with recurrent subluxation or dislocation but no history of significant trauma. Assessment for instability includes apprehension sign (externally rotate the abducted arm—feeling of impending dislocation suggests anterior instability); sulcus sign (provide inferior traction on relaxed arm—increased sulcus between humerus and acromion suggests inferior instability); jerk test (flex and internally rotate the arm, then flex the elbow; push the humerus posteriorly—subluxation or dislocation confirms posterior instability). Treatment for shoulder instability includes strengthening of rotator cuff and scapular stabilizer muscles; orthotics/braces to maintain anatomic alignment; or where conservative treatment fails, surgery to tighten or repair capsular structures and ligaments. 2 2. E [23, 24] The anterior portion of the disc is primarily supplied by the gray rami communicans after it emerges from the sympathetic chain joining the spinal segmental nerve. The posterior disc is primarily supplied by the posterior longitudinal ligament plexus which receives the majority of its input from the sinuvertebral nerve. The posterior primary ramus branches into medial and lateral branches. The former of these supplies the dorsal spinal column components. The annulus fibrosis therefore has contributions from all of the above.

References 1. Ben-David R.  Complications of neuraxial blockade. Anesthesiol Clin North Am. 2002;20:669–93, 675. 2. Kandel ER, Schwartz JH, Jessell TM.  Principles of neural science. 3rd ed. New  York: McGraw-Hill; 2000. p. 1313.

2 Anatomy 3. Lawrence R.  Thieme atlas of anatomy. 470, fig b. New York: Thieme Verlag; 2006. 4. Patt R, Plancarte R.  Superior hypogastric plexus block: a new therapeutic approach for pelvis pain. In: Waldman SD, Winnie AP, editors. Interventional pain management. 1st ed. Philadelphia: WB Saunders; 1996. p. 384–91. 5. Raj PP.  Stellate ganglion block. In: Waldman SD, Winnie AP, editors. Interventional pain management. Philadelphia: WB Saunders; 1996. p. 384–91. 6. Dumitru D, Zwarts MJ, Amato AA, editors. Electrodiagnostic medicine. Philadelphia, PA: Hanley and Belfus, Inc.; 2002. p. 780. 7. Johna S. Gray’s anatomy: the anatomical basis of clinical practice. 39th ed. New York: C.V. Mosby; 2004. 8. Smith LK, Weiss EL, Lehmkuhl LD. Brunnstrom’s clinical kinesiology. Philadelphia: FA Davis; 1996. p. 374. 9. Waldman SD.  Greater and lesser occipital nerve blocks. In: Waldman SD, editor. Atlas of interventional pain management. 2nd ed. Philadelphia: Saunders; 2004. p. 23. 10. McDonald J. Obstetric pain. In: Wall and Melzack’s textbook of pain. 5th ed. p. 794–5. 11. Bonica JJ. Autonomic innervation of the viscera in relation to nerve block. Anesthesiology. 1968 Jul-Aug; 29(4):793–813. https://doi.org/10.1097/00000542196807000-00023. PMID: 4874158. 12. Kawatani M, de Groat WC. A large proportion of afferent neurons innervating the uterine cervix of the cat contain VIP and other neuropeptides. Cell Tissue Res. 1991 Oct; 266(1):191–6. https://doi.org/10.1007/ BF00678724. PMID: 1721005. 13. White AA, Panjabe MM. Biomechanics of the lumbar spine. 2nd ed. Philadelphia: Lippincott; 1990. p. 13–6. 14. Towney LT, Taylor JR.  Physical therapy of the low back. New York: Churchill-Livingstone; 1994. p. 70. 15. Stoller DW.  Stolle’s atlas of orthopedic and sports medicine. Wolters-Kluwer; 2008. p. 552. 16. Bogduk N.  Clinical anatomy of the lumbar spine and sacrum, vol. 1997. 3rd ed. New York: Churchill-­ Livingstone. p. 7. 17. Czervionke LF, Fenton DS. Image guided spine intervention. Philadelphia: Saunders; 2003. p. 23. 18. Field D.  Anatomy, palpation, and surface markings. Butterworth; 1999. p. 128. 19. Gray H.  Anatomy of the human body. Philadelphia: Lea & Febiger; 1918. Bartleby.com. 2000. www.bartleby.com/107/. 20. Hoppenfled S. Physical examination of the spine and extremities. New York: Appleton & Lange. p. 82. 21. Wirth MA.  Shoulder. In: Greene WB, editor. Essentials of musculoskeletal care. 2nd ed. Rosemont, IL: AAOS; 2001. p. 107–14. 22. Cuccurullo SJ, editor. Physical medicine and rehabilitation board review. New York: Demos; 2004. p. 141. 23. Bogduk N, Tynan W, Wilson AS. The nerve supply to the intervertebral discs. J Anat. 1981;132:39–56. 24. Malik K, Nelson A. Overview of low back pain disorders. In: Benzon H, editor. Essentials of pain medicine. 4th ed: Elsevier Health Sciences. p. 2018.

3

Neurochemistry and Pain Processing

Questions 1. Which chemical is released in response to platelet-activating factor derived from mast cell degranulation leading to pain A. Histamine B. Bradykinin C. Histamine D. H+ 2. Which of the following cytokines directly excites and sensitizes nociceptive afferent fibers to both thermal and mechanical stimuli A. IL-6 B. TNF-alpha C. IL-10 D. Trypsin 3. Opioids bind to cells via which type of membrane receptor? A. G protein-coupled receptor B. Ion channel linked receptor C. Enzyme-linked receptor D. Ligand-gated ion receptor 4. Which of the following is the primary excitatory neurotransmitter in the somatosensory system? A. Glycine B. GABA-A C. ATP D. Glutamate 5. A 48-year-old male is found to be unresponsive in his car at 2 am. He is found by

paramedics and given naloxone without regaining responsiveness. He is rushed to the emergency room and toxicity screening reveals high levels of alcohol and diazepam in his system. Alcohol and diazepam mediate their effects through: A. Glycine receptor B. GABA-B receptor C. GABA-A receptor D. Serotonin expression 6. Serotonin chiefly expresses its descending inhibition to the spinal dorsal horns through pathways originating in A. Limbic system B. Midbrain raphe nucleus C. Ventromedial nucleus of the hypothalamus D. Orbitofrontal cortex 7. An 82-year-old woman suffered from a blistering, painful rash on her left flank approximately 4  months ago. Her blisters have resolved but she has continued to have burning pain in the same distribution as the original rashes since that time. She has tried multiple oral medications as well as topical lidocaine without significant relief of her pain. You decide to trial a capsaicin patch. Which of the following receptors is responsible for the pain felt with this treatment modality? A. ASIC B. TRPV1

© The Author(s), under exclusive license to Springer Nature Switzerland AG 2021 M. Suer, N. Sehgal, Questions and Answers in Pain Medicine, https://doi.org/10.1007/978-3-030-68204-0_3

15

16

3  Neurochemistry and Pain Processing

C. TRPM D. The peri-aquiductal gray has been shown D. TRPV2 to be involved experimentally 8. Which of the following endogenous opioids E. A and B does NOT bind to the mu receptor? F. C and D A. Endorphins 1 3. A 43-year-old male is seen in follow up at B. Endomorphins the pain clinic. Multiple treatments have C. Morphiceptin been tried to treat his pain over the last sev D. Dynorphins eral years including injections, topical medi 9. A 21-year-old male is found unresponsive cations, oral medications, and intravenous at a party at a local college. The paramedics infusions. However, his pain has remained arrive at the scene and are told the patient without decrement despite all of your interlooks to be apneic. They administer naloxventions. In clinic today, he reports having one and the patient has return of spontanesmoked marijuana on multiple occasions ous breathing. Rank the opioid receptors by with improvement in his pain. What receporder of effect on respiratory drive. tor, found primarily in the dorsal root gan A. Delta > mu > kappa glion, is suspected to be responsible for the B. Mu > kappa > delta pain-relieving effects of cannabinoids? C. Mu > delta > kappa A. CB1 D. Mu > delta = kappa B. CB2 10. In the above scenario, naloxone acts on C. NK1 which receptor: D. Mu A. Mu 14. During an action potential, which of the fol B. Delta lowing best describes ion flow C. Kappa A. Na+ flows into the cell resulting in depo D. All of the above larization, then K+ flows into the cell 11. In individuals without chronic pain, the NK1 resulting in repolarization receptor is found most commonly in _____. B. Na+ flows into the cell resulting in depoIn central sensitization, there is internalizalarization, then K+ flows out of the cell tion of the NK1 receptor in _______. resulting in repolarization A. Lamina I; lamina II C. K+ flows into the cell resulting in depo B. Lamina IV; Lamina I larization, then Na+ flows out of the cell C. Lamina II; Lamina III and IV resulting in repolarization D. Lamina I; Lamina III and IV D. Ca+ flows into the cell resulting in depo12. A 39-year-old female with chronic pain larization, then Na+ flows out of the cell is enrolled in a double blind, placebo-­ resulting in repolarization controlled study of a new pain medication. 15. A 43-year-old male is seen in follow up at Unbeknownst to her, she is enrolled in the the pain clinic. Multiple treatments have placebo arm of the study. However, she dembeen tried to treat his pain over the last onstrates improved pain scores on follow up several years including injections, topical visits. Which of the following is true of the medications, oral medications, intravenous placebo response to pain? infusions, and intrathecal morphine. You A. There is no scientific basis decide to trial intrathecal Ziconotide, which B. Functional imaging has failed to show acts on: cerebral changes in response to noxious A. Sodium channels stimuli B. L-type voltage gated calcium channels C. Endogenous opioid changes have been C. N-type voltage gated calcium channels demonstrated D. Mu1 opioid receptor

Answers

16. Which of the following is NOT true regarding the NDMA receptor A. Glycine and glutamate bind to the NR1 and NR2 subunits resulting in activation B. Magnesium and Zinc can bind to the receptor blocking other cations C. Activation of the receptor results in removal of the Magnesium ion allowing efflux of Calcium ions D. Memantine is an uncompetitive NMDA channel blocker

Answers 1. A [1] A. Correct—Histamine B. Bradykinin is released by kininogens and produces acute pain in humans by activation of unmyelinated and myelinated nociceptors C. Histamine is released from mast cells by substance P and CGRP.  Exogenous histamine applied to the skin produces itch response by not pain but does potentiate the responses of nociceptors to bradykinin and heat D. Low pH contributes to pain and hyperalgesia associated with inflammation 2. B [1] A. IL-6 sensitizes nociceptors to heat B. Correct—TNF-alpha C. IL-10 has shown anti-nociceptive properties D. Trypsin, a proteinase, is not tradition ally considered part of the inflammatory soup, but may act on proteinase-activated receptors 3. A [2] A. Correct—G protein-coupled receptor B. Ion channel linked receptor is incorrect C. Enzyme-linked receptor is incorrect D. Ligand-gated ion receptor is another name for ion channel linked receptor 4. D [3] A. Glycine is an inhibitory neurotransmitter B. GABA-A is an inhibitory neurotransmitter

17

C. ATP is excitatory and modulates somatosensory transmission. Receptors of ATP are found in lamina II and V of the dorsal horn where they increase the release of glutamate D. Correct—Glutamate 5. C [1] Alcohol, diazepam, and barbiturate’s effects are mediated through the GABA-A receptor which is linked to a chloride channel. 6. B [4] A. The limbic system is involved in emotion, behavior, motivation, and long-term memory B. Correct—Midbrain raphe nucleus C. VMH is involved with hunger (particularly satiety), fear, sexual activity, and thermoregulation. D. A region in the frontal lobe associated with cognitive processing of decisionmaking 7. B [5] A. H+ is the optimal stimulus for ASIC (acid sensing ion channel) B. Correct. TRPV1 receptors respond to temperatures greater than 43 °C and capsaicin C. TRPM typically responds to temperatures of 25–28 °C as well as menthol D. TRPV2 typically responds to temperatures greater than 52 °C 8. D [6] All of the answers bind to the mu opioid receptor except dynorphins, which bind to the kappa receptor. The enkephalins bind to the delta and, to a lesser extent, the mu receptor 9. C [7] Of the opioid receptors, agonism of the mu and delta receptors is most associated with respiratory drive. Centrally in control of respiration, these are found in the ventrolateral medulla and the dorsolateral pons. 10. D [8] Naloxone, a pure non-selective competitive opioid receptor agonist, works on all three opioid receptors with the highest affinity for mu, followed by delta and kappa.

18

11. D [9] NK1, the receptor of substance P, is found with highest concentration in lamina 1 of the spinal cord. Noxious stimuli induce internalization of these receptors from the cell surface into intracellular compartments in layers 1, 3, and 4. This is significant as layers 3 and 4 are not classically responsive to noxious stimuli. 12. F [10–12] Both C and D are true. Multiple groups have demonstrated the medial prefrontal/ rostral ACC responses to placebo seem to recruit the periaqueductal gray (PAG) and amygdala and involvement of the PAG in placebo-­induced analgesia is experimentally observed as well, linking the opioid descending modulation with prefrontal cortical control of placebo analgesia. 13. A [1] A. CB1 receptor activation leads to decrease in pain transmission. These receptors are expressed highly in the dorsal root ganglion. B. More associated with immune function, CB2 receptors are not expressed highly in the dorsal root ganglion. C. NK1 is the receptor for substance P D. Mu is an opioid receptor 14. B Na+ flows into the cell resulting in depolarization, then K+ flows out of the cell resulting in repolarization. 15. C [13] Ziconotide, derived from the venom of the Conus magus snail, reversibly blocks the N-type voltage-sensitive calcium channels. These channels are found at presynaptic nerve terminals in the dorsal horn region of the spinal cord mostly in Rexed laminae I and II. 16. C [1, 14] Activation of the NMDA receptor results in calcium influx, not efflux. The other answer choices are correct. The NMDA receptor is only activated by intense or prolonged stimuli which can remove the magnesium ion that blocks the channel.

3  Neurochemistry and Pain Processing

References 1. Nouri KH, et  al. Neurochemistry of somatosensory and pain processing. In: Benzon HT, et  al., editors. Essentials of pain medicine. Elsevier; 2018. 2. Loh HH, Smith AP.  Molecular characterization of opioid receptors. Annu Rev Pharmacol Toxicol. 1990;30:123–47. https://doi.org/10.1146/annurev. pa.30.040190.001011. PMID: 2160790. 3. Sheffler ZM, Reddy V, Pillarisetty LS.  Physiology, neurotransmitters. [Updated 21 May 2020]. In: StatPearls [Internet]. Treasure Island, FL: StatPearls Publishing; 2020. https://www.ncbi.nlm.nih.gov/ books/NBK539894/. 4. Walker EP, Tadi P.  Neuroanatomy, nucleus raphe. [Updated 10 Jul 2020]. In: StatPearls [Internet]. Treasure Island, FL: StatPearls Publishing; 2020. https://www.ncbi.nlm.nih.gov/books/NBK544359/. 5. Nagy I, Friston D, Valente JS.  Pharmacology of the capsaicin receptor, transient receptor potential vanilloid type-1 ion channel. Prog Drug Res. 2014;68:39–76. 6. Chavkin C, James IF, Goldstein A.  Dynorphin is a specific endogenous ligand of the kappa opioid receptor. Science. 1982;215(4531):413–5. https://doi. org/10.1126/science.6120570. PMID: 6120570. 7. Pattinson KT.  Opioids and the control of respiration. Br J Anaesth. 2008;100(6):747–58. https://doi. org/10.1093/bja/aen094. Epub 2008 May 1. PMID: 18456641. 8. Jordan MR, Morrisonponce D.  Naloxone. [Updated 10 Jul 2020]. In: StatPearls [Internet]. Treasure Island, FL: StatPearls Publishing; 2020. https://www. ncbi.nlm.nih.gov/books/NBK441910/. 9. Bowden JJ, Garland AM, Baluk P, et al. Direct observation of substance P-induced internalization of neurokinin 1 (NK1) receptors at sites of inflammation. Proc Natl Acad Sci U S A. 1994;91(19):8964–8. https://doi.org/10.1073/pnas.91.19.8964. 10. Pecina M, Zubieta JK.  Molecular mechanisms of placebo responses in humans. Mol Psychiatry. 2015;20:416–23. 11. Scott DJ, Stohler CS, Egnatuk CM, Wang H, Koeppe RA, Zubieta JK. Placebo and nocebo effects are defined by opposite opioid and dopaminergic responses. Arch Gen Psychiatry. 2008;65:220–31. 12. Fingkamp M, Dougherty PM, Raja SN. Anatomy and physiology of the pain signaling process. In: Benzon HT, et al. Essentials of pain medicine. Elsevier; 2018. 13. Ziconotide. 2020. www.pdr.net/drug-­summary/ Prialt-­ziconotide-­782.1142#15. 14. Jewett BE, Thapa B.  Physiology, NMDA recep tor. [Updated 3 Sep 2020]. In: StatPearls [Internet]. Treasure Island, FL: StatPearls Publishing; 2020. https://www.ncbi.nlm.nih.gov/books/NBK519495/.

4

Pain Signaling Processes

Questions 1. Rank the following sensory nerve fiber types by conduction velocity (fastest to slowest) A. A-delta > A-beta > C B. C > A-delta > A-beta C. A-beta > A-delta > C D. A-beta = A-delta > C 2. A 56-year-old patient with diabetes mellitus, type 2, presents for evaluation of low back pain. In testing of peripheral sensation, you utilize a swath of cotton to touch the bottom of their foot. What sensory nerve fibers are being activated for sensation? A. A-alpha B. A-beta C. A-delta D. C 3. A 4-year-old is walking around his kitchen while his father is cooking dinner. He reaches onto the hot stovetop and burns his hand. Which sensory nerve fibers are responsible for the immediate pain felt? A. A-alpha B. A-beta C. A-delta D. C 4. Which nerve fibers are responsible for the slower-onset burning sensation following the brief intense heat stimulus? A. A-alpha B. A-beta

C. A-delta D. C 5. Activity of this afferent with high mechanical and low heat threshold almost certainly mediates the “first” acute pain response to noxious heat A. A-alpha B. A-beta C. A-delta D. C 6. Which of the following Rexed lamina are located in the dorsal horn of the spinal cord gray matter? A. I–XII B. I–VI C. I–IX D. III–VIII 7. Which of the following is not considered a medial pain pathway? A. Spinoamygdalar B. Spinoreticular C. Spinothalamic D. Medial spinothalamic 8. Which of the following Rexed lamina is also known as the marginal nucleus? A. I B. II C. III D. IV 9. After hitting a home run in his first at bat, Randal Acunar is struck on the elbow by a 97  mph fastball. When he reaches base, he

© The Author(s), under exclusive license to Springer Nature Switzerland AG 2021 M. Suer, N. Sehgal, Questions and Answers in Pain Medicine, https://doi.org/10.1007/978-3-030-68204-0_4

19

20

continues to have lingering pain where the ball struck his elbow. Sensory afferents relaying this prolonged, less intense pain most likely terminate at which Rexed lamina? A. I B. III C. IV D. V 10. Which of the following second order neurons when provided with an increase stimulus increase the magnitude or intensity of their propagated action potential. A. Wide Dynamic Range B. Nociceptive Specific C. Gracile fasciculus D. Cuneate fasciculus 11. Which type of ion channel is up-regulated in chronic pain states resulting in ongoing pain from ectopic activity and serves as a target for current FDA-approved pain medications? A. Potassium B. Magnesium C. Calcium D. Sodium 12. A 34-year-old female presents with chronic pain in her right arm. You repeated tap her hand one time per second, to which she notes increased feeling of pain with repeated tapping. This phenomenon is associated with which of the following? A. A-delta afferent fibers B. A-beta afferent fibers C. Lamina V—WDR neurons D. Nociceptive-specific neurons 13. Which of the following brain centers has not been shown to be associated with acute and chronic pain states in imaging studies? A. Insula B. Prefrontal cortex C. Thalamus D. Occipital lobe 14. Which of the following does not describe asymbolia A. Lack of or inappropriate emotional response to painful stimuli B. A type of aphasia

4  Pain Signaling Processes

C. Can be caused by ischemic damage to the insular cortex D. Has a strong association with the placebo effect 15. Descending modulation of nociception can have both inhibitory and facilitatory effects on neurons in the dorsal horn. Which of the following regions can facilitate both of these effects A. Rostral ventromedial medulla B. Periaqueductal grey C. Rostral ventrolateral medulla D. Periaqueductal pink 16. The processes resulting in noxious stimulus-­ inducing pain are all of the following except: A. Transduction B. Transition C. Modulation D. Perception 17. Which of the following is the primary pain-­ signaling channel of the somatosensory system A. Anterolateral system B. Gracile fasciculus C. Cuneate fasciculus D. Anteromedial system 18. Spinothalamic tract neurons in deeper laminae send their axons through the ventrolateral spinal white matter on the opposite side and through the ventrolateral brainstem to A. Ventral posterolateral nucleus of the thalamus B. Thalamic reticular nucleus C. Ventral posteromedial nucleus of the thalamus D. Occipital lobe 19. Descending pathways to the spinal cord that modulate pain include axons originating in A. Medullary reticular formation B. Pedunculopontine nucleus C. Corpora quadrigemina D. Occipital lobe 20. Which of the following medications produces its effects upon the GABA-B receptor? A. Gabapentin B. Pregabalin C. Baclofen D. Tizanidine

Questions

21. Which afferent fibers are involved in the development of central sensitization in the dorsal horn? A. A beta fibers B. C fibers C. A alpha fibers D. A delta fibers E. Gamma fibers 22. A correct statement regarding autonomic afferent and efferent pathways is A. Afferent fiber cell bodies are located in the dorsal root ganglion B. Preganglionic efferent axons are unmyelinated slow conducting C fibers C. Postganglionic efferent neurons are located within the CNS D. Each preganglionic axon synapses with a single postganglionic neuron E. Parasympathetic outflow occurs via thoracodorsal fibers 23. Which of the following is true of the sympathetic nervous system? 1. Preganglionic neurons are located in the T1 to L3 spinal segments 2. The sympathetic chain contains postganglionic neurons 3. Some preganglionic fibers end on ganglia located close to the viscera 4. Norepinephrine is released by sympa thetic preganglionic endings A. 1 and 3 B. 2 and 4 C. 1, 2 and 3 D. 4 only E. all of the above 24. Acetylcholine is the neurotransmitter at the following nerve endings: A. All preganglionic neurons B. Parasympathetic postganglionic neurons C. Sympathetic postganglionic neurons to sweat glands D. Vasodilator nerve endings on skeletal muscle blood vessels E. All of the above 25. Which of the following sensory receptors are absent in the viscera?

21

A. Proprioceptors B. Osmoreceptors C. Chemoreceptors D. Baroreceptors E. Thermoreceptors 26. Nerve fibers differ in their susceptibility to conduction blocks induced by various methods. Select the correct statement regarding the relative susceptibility (from most susceptible to least susceptible) of A, B and C nerve fibers to A. Hypoxia: A fiber > B fiber > C fiber B. Pressure: B fiber > A fiber > C fiber C. Local anesthetic: C fiber  >  B fiber  >  A fiber D. Pressure: C fiber > B fiber > A fiber 27. In the periphery, pain impulses normally travel along A. Rapidly conducting large diameter A α (alpha) nerve fibers B. Myelinated A ß (beta) fibers conducting at 50 m/s C. Thickly myelinated A γ (gamma) fibers conducting at 10 m/s D. Thinly myelinated A δ (delta) fibers conducting at 20 m/s E. Myelinated C fibers conducting at 30 m/s 28. Nociceptors are a specialized class of primary afferents that respond to intense noxious stimuli. Which of the following is true of nociceptors? A. Nociceptors are specialized terminations of A-beta fibers B. Pain due to chemical stimuli is mediated exclusively by A fibers C. Nociceptor responsiveness is decreased after tissue injury D. Locally released inflammatory mediators sensitize nociceptors E. Primary hyperalgesia is due to neuronal sensitization in the CNS 29. Name the inhibitory neurotransmitter A. Glutamate B. GABA C. Substance P D. Bradykinin

22

Answers 1. C [1] The fastest-conducting fibers are the largediameter myelinated A beta (Aβ) fibers at >20 m/s transmitting light touch, pressure, or hair movement. The axons of nociceptive neurons are generally thinly myelinated A delta (Aδ) fibers (2–20 m/s) or unmyelinated C fibers ( A > C   Pressure: A > B > C   Local anesthetics: C > B > A 27. D [27] Nerve fibers are classified based on fiber diameter, conduction velocity, origin and function. Fiber diameter and conduction velocity are determined by the degree of myelination. Large-diameter fibers are

25

thickly myelinated and rapidly conducting. Small-diameter fibers are thinly myelinated and conduct relatively slowly. C fibers are unmyelinated and conduct very slowly. 28. D [28] Nociceptors are free nerve endings of thinly myelinated A and unmyelinated C fibers. C fiber afferents signal burning pain from intense heat stimuli, as well as pain from sustained pressure. A fiber afferents signal sharp pain from heat stimuli and sharp mechanical stimuli. Both A and C fiber nociceptors signal pain from chemical stimuli. Following a cutaneous injury enhanced pain in response to cutaneous stimuli develops at the site of injury (primary hyperalgesia) and in the surrounding uninjured skin (secondary hyperalgesia). Primary hyperalgesia results from enhanced responsiveness of nociceptors (sensitization) and is due to local release of inflammatory mediators after tissue injury. Secondary hyperalgesia is due to sensitization of neurons in the central nervous system. 2 9. B [29] All of the neurotransmitters mentioned are excitatory except for GABA.

References 1. Burgess PR, Perl ER.  Myelinated afferent fibres responding specifically to noxious stimulation of the skin. J Physiol. 1967;190:541–62. 2. Meyer RA, Campbell JN.  Myelinated nociceptive afferents account for the hyperalgesia that follows a burn to the hand. Science. 1981;213:1527–9. 3. Ringkamp M, Dougherty PM, and Raja S.  Anatomy and physiology of the pain signaling process. In: Benzon HT, et al., editors. Essentials of pain medicine. Elsevier; 2018. 4. 1962-, Striedter GF.  Neurobiology: a functional approach (Instructor’s ed.). New  York: Oxford University Press; 2016. 5. Rexed B. The cytoarchitectonic organization of the spinal cord in the cat. J Comp Neurol. 1952;96(3):414–95. 6. Rexed B. The cytoarchitectonic organization of the spinal cord in the cat. J Comp Neurol. 1952;96:415–95. 7. D’Mello R, Dickenson AH. Spinal cord mechanisms of pain. Br J Anaesth. 2008;101(1):8–16. 8. Basbaum AI, Bautista DM, Scherrer G, Julius D. Cellular and molecular mechanisms of pain. Cell. 2009;139:267–84.

26 9. Darby SA, Cramer GD. Clinical anatomy of the spine, spinal cord and ANS. Saint Louis, MO: Mosby; 2013. p. 341–413. 10. Mendell LM. Computational functions of neurons and circuits signaling injury: relationship to pain behavior. Proc Natl Acad Sci U S A. 2011;108:15596–601. 11. Berthier M, Starkstein S, Leiguarda R. Asymbolia for pain: a sensory-limbic disconnection syndrome. Ann Neurol. 1988;24:41–9. 12. Helmstetter FJ, Tershner SA, Poore LH, Bellgowan PS.  Antinociception following opioid stimulation of the basolateral amygdala is expressed through the periaqueductal gray and rostral ventromedial medulla. Brain Res. 1998;779:104–18. 13. Saade NE, Jabbur SJ.  Nociceptive behavior in animal models for peripheral neuropathy: spinal and supraspinal mechanisms. Prog Neurobiol. 2008;86: 22–47. 14. Fields HL, Malick A, Burstein R. Dorsal horn projection targets of ON and OFF cells in the rostral ventromedial medulla. J Neurophysiol. 1995;74:1742–59. 15. Institute of Medicine (US) Committee on Pain, Disability, and Chronic Illness Behavior; Osterweis M, Kleinman A, Mechanic D, editors. Pain and disability: clinical, behavioral, and public policy perspectives. 7. The anatomy and physiology of pain. Washington, DC: National Academies Press; 1987. https://www.ncbi.nlm.nih.gov/books/NBK219252/. 16. Mendoza JE.  Anterolateral system. In: Kreutzer JS, DeLuca J, Caplan B, editors. Encyclopedia of clinical neuropsychology. New York: Springer; 2011. https:// doi.org/10.1007/978-­0-­387-­79948-­3_704. 17. Giuffrida R, Rustioni A. Dorsal root ganglion neurons projecting to the dorsal column nuclei of rats. J Comp Neurol. 1992;316(2):206–20. 18. Al-Chalabi M, Reddy V, Gupta S.  Neuroanatomy, spinothalamic tract. [Updated 12 Aug 2020]. In: StatPearls [Internet]. Treasure Island, FL: StatPearls

4  Pain Signaling Processes Publishing; 2020. https://www.ncbi.nlm.nih.gov/ books/NBK507824/. 19. Kiernan J, Rajakumar R.  Barr’s the human nervous system: an anatomical viewpoint. Philadelphia: Lippincott Williams & Wilkins; 2013. 20. Mangold SA, Das JM. Neuroanatomy, reticular formation. [Updated 15 Aug 2020]. In: StatPearls [Internet]. Treasure Island, FL: StatPearls Publishing; 2020. https://www.ncbi.nlm.nih.gov/books/NBK556102/. 21. Tsang EW, Hamani C, Moro E, Mazzella F, Poon YY, Lozano AM, Chen R. Involvement of the human pedunculopontine nucleus region in voluntary movements. Neurology. 2010;75(11):950–9. 22. Marieb EN. Essentials of human anatomy & physiology. 6th ed. San Francisco: Daryl Fox; 2000. p. 210. 23. Ghanavatian S, Derian A. Baclofen. [Updated 22 Aug 2020]. In: StatPearls [Internet]. Treasure Island, FL: StatPearls Publishing; 2020. https://www.ncbi.nlm. nih.gov/books/NBK526037/. 24. Campbell JN, Meyer RA. Mechanisms of neuropathic pain. Neuron. 2006;52:77–92. 25. Ganong WF.  The autonomic nervous system. In: Review of medical physiology. 22nd ed. New  York: McGraw-Hill; 2005. p. 223. 26. Ganong WF.  Cutaneous, deep and visceral sensa tion. In: Review of medical physiology. 22nd ed. New York: McGraw-Hill; 2005. p. 1433. 27. Ganong WF.  Excitable tissue: nerve. In: Review of medical physiology. 22nd ed. New  York: McGraw-­ Hill; 2005. p. 60–1. 28. Meyer RA, et  al. Peripheral mechanisms of cutaneous nociception. In: Mc Mahon SB, Koltzenburg M, editors. Wall & Melzack textbook of pain. 5th ed. Philadelphia: Elsevier/Churchill Livingstone; 2006. p. 3. 29. Shiah IS, Yatham LN.  GABA function in mood disorders: an update and critical review. Life Sci. 1998;63(15):1289–303.

5

Physical Examination and Assessment

Questions 1. A 38-year-old female is referred to the pain clinic for neck pain with intermittent radiation of pain into her hands. On neurologic evaluation, you find no deficits in sensation in the C4–T1 dermatomes to light touch, pinprick, or cold. Strength testing reveals no deficits in her upper or lower extremities. Cerebellar testing is normal as well. On gait testing, she has no abnormalities noted with normal gait. However, with increased pace, you notice her feet each cross over the midline. What term best describes this abnormal gait pattern? A. Scissoring B. Steppage C. Circumduction D. Antalgic 2. The above gait pattern can be seen in all of the following conditions EXCEPT: A. Cerebral palsy B. Cervical myelopathy C. Multiple Sclerosis D. Gluteus medius weakness 3. On further examination of the above female, you perform passive range of motion exercises of her lower extremities. With slow, controlled movements, you not no changes in her tone or in the range of motion in her knees with flexion or extension. When you

increase the speed of testing, you find that she has a small catch with both flexion and extension with no resistance throughout the remainder of the range of motion. This is best described as: A. Dystonia B. Spasticity, Grade 2 Modified Ashworth Scale C. Spasticity, Grade 1 Modified Ashworth Scale D. Guarding 4. A 22-year-old female presents to the pain clinic for initial evaluation of chronic headaches. She describes unilateral headaches that have a pulsating quality occurring on a weekly basis. She describes both avoidance of light and noise during these episodes. Which of the following maneuvers of cranial nerve testing is appropriately matched with its respective cranial nerve: A. Rinne test—sensorineural hearing B. Extraocular movements—CN III, V, VI C. Medial scapular winging—CN XI D. Jaw opening—CN V 5. A 56-year-old male with acute-onset low back pain presents for initial evaluation after injuring his back shoveling show 3  months prior. His pain diagram reveals aching pain in the low back radiating down the lateral aspect of his right leg described as shooting and stabbing. He has decrement of his hamstring reflex

© The Author(s), under exclusive license to Springer Nature Switzerland AG 2021 M. Suer, N. Sehgal, Questions and Answers in Pain Medicine, https://doi.org/10.1007/978-3-030-68204-0_5

27

28

5  Physical Examination and Assessment

on the right. Which of the ­following would cramping in his legs with upright ambulation most likely account for his presentation relieved with sitting or leaning forward and A. L4–L5 right paramedian disc bulge on has evidence of severe spinal stenosis at MRI L4–5 on MRI. On exam, you note no strength B. Severe degenerative changes with spinal deficits in his upper or lower extremities. He stenosis noted at L3–4, L4–5, and L5–S1 has 1+ symmetric reflexes at the patella and C. L5–S1 right paramedian disc bulge on Achilles with upgoing Babinski bilaterally. MRI Upper extremity reflexes are 2+ at the biceps D. Grade 1 degenerative spondylolisthesis and brachioradialis and Hoffman’s is posiof L4 on L5 without evidence of central tive bilaterally. What is the next step in the canal stenosis management of this patient. 6. A 78-year-old woman with a history of A. Referral for surgical intervention chronic neck pain without radiation is evalu B. Interlaminar ESI at L4–5 ated in the pain clinic. In manual muscle test C. Interlaminar ESI at L3–4 ing, you have her spread all of her fingers out D. Cervical MRI (abduct) while you apply pressure to her 1 0. You are evaluating a 28-year-old woman for pinky and first fingers to attempt to adduct jaw pain and headaches. On exam, you tap the fingers together. This evaluates which of over the angle of the mandible and note a the following pathways: spasm of the masseter muscle. This is most A. Median nerve likely indicative of which of the following: B. Ulnar nerve A. Hyperkalemia C. C6–8 nerve roots B. Hypocalcemia D. C7–8 nerve roots C. Masseter trigger point 7. In performance of sensory testing of the D. TMJ syndrome upper extremity dermatomes, which of the 11. The physical exam test with the greatest following is INCORRECTLY matched: diagnostic value for Thoracic Outlet A. C6 dermatome—dorsum of digit 2 Syndrome is: B. Radial nerve—Dorsal finger tips of digits A. Adson’s 1–3 B. Roos C. T1 dermatome—medial epicondyle of C. Tinel’s the elbow D. Phalen’s D. C7 dermatome—dorsum of the digit 3 E. Spurling’s 8. A 43-year-old male presents with low back 12. A 78-year-old obese female presents with pain. He has radiation of the pain into the pain in the right buttock, groin and thigh. She buttocks and, intermittently, into the upper has no pain at rest or while sitting. She has hamstrings. Chiropractic care has been pain on walking, and finds that if she uses a attempted with temporary relief of his pain. cane in her left hand to support herself, she You suspect he has either facet- or sacroiliac-­ can walk with less pain. On examination she mediated pain. Which of the following tests has a Trendelenburg gait, moderately limited provides the highest sensitivity of identifylumbar flexion and extension, pain limited ing sacroiliac joint pain: hip movements, pain provocation on external A. Pain mapping of unilateral buttock pain rotation of the hip, tenderness in the right below the level of L5 without midline pain groin, buttock, and greater trochanter. B. Gillet Test (Marching Test) Neurologic examination reveals negative C. Laslett’s Cluster straight leg raise, antalgic weakness of hip D. Centralization flexors and knee extensors, normal sensation 9. You evaluate a 64-year-old man with chronic and reflexes. This examination suggests that low back pain. He reports dysesthesias and the most likely source of pain is the

Answers

. Lower lumbar facet joint A B. Sacro-iliac joint C. Hip joint D. Piriformis muscle E. Trochanteric bursa 13. What factors commonly interfere with the accurate assessment of chronic pain? A. Subjective nature of pain B. Culture C. Linguistic patterns D. Ethnicity E. All of the above

Answers 1. A [1] A. Crossing of the legs while advancing with slow, small steps. Seen with CVA, SCI, syringomyelia, MS, liver failure, cervical myelopathy, cerebral palsy. B. Caused by foot drop, so toes scrape the ground. Seen with lumbar HNP, peroneal atrophy, peroneal neuropathy or polyneuropathy, Guillain-Barré, MS, SCI, poliomyelitis. C. Swing limb is advanced in a semicircular arc to create extra clearance when the functional length of the advancing limb cannot be shortened in the usual fashion with hip flexion, knee flexion, or ankle dorsiflexion. D. Avoidance of weight bearing on the affected limb due to pain (i.e., short stance phase). Seen in patients with radicular pain or painful lower extremity conditions, such as hip or knee osteoarthritis or other trauma to the affected limb. 2. D [1] Scissoring gait can be seen in multiple conditions including: CVA, SCI, syringomyelia, MS, liver failure, cervical myelopathy, cerebral palsy. Gluteus medius typically results in a Trendelenburg (can be modified Trendelenburg) gait. 3. C [1, 2] A. Dystonia is a state of abnormal muscle tone resulting in muscular spasm and

29

abnormal posture. Dystonia is NOT dependent upon velocity of movement, whereas spasticity is dependent on velocity of movement B. Modified Ashworth Scale scoring • 0—No increase in tone • 1—Slight increase in muscle tone, manifested by a catch and release or minimal resistance at end ROM when affected part is moved in flexion or extension • 1+—slight increase in muscle tone, manifested by a catch, followed by minimal resistance throughout the remainder (less than half) of the ROM • 2—More marked increase in muscle tone through most of the range of movement, but affected part easily moved • 3—Considerable increase in muscle tone, passive movement difficult • 4—affect part rigid in flexion or extension C. See above D. Guarding, or patient-induced resistance to movement, would be more characterized by pain with range and inability for the patient to relax the muscles. You would not have a catch and release pattern with this. 4. D [3] A. Rinne test measures bone vs acoustic conduction whereas Weber test evaluates sensorineural hearing loss B. Eye movements are controlled by CN III, IV (superior oblique), and VI (lateral rectus) C. CN XI (spinal accessory) palsy would result in lateral winging, whereas long thoracic palsy (serratus anterior muscle) would result in medial winging D. CN V controls facial sensation and muscles of mastication 5. A [4] A. L4–L5 right paramedian disc bulge on MRI would most likely contact the traversing right L5 nerve root resulting in the above presentation of an L5 radiculopathy

30

B. Severe degenerative changes with spinal stenosis noted at L3–4, L4–5, and L5–S1 would not result in the above focal radicular pattern without focal protrusion contacting the right L5 nerve root. Findings in this patient would be more consistent with neurogenic claudication vs degenerative arthropathy C. L5–S1 right paramedian disc bulge on MRI would result in impingement of the right S1 nerve root as the right L5 nerve root exits high in the neuroforamen prior to the disc bulge at L5–S1. D. Degenerative spondylolisthesis would not result in the focal nature of the above radiculopathy would evidence of other pathology noted on MRI. 6. B [5] Finger abduction is controlled by the ulnar nerve (roots C8–T1). The median nerve can be assessed in the hand by thumb palmar abduction or flexion of thumb IP and index DIP (as in making the “OK” sign). Radial nerve (C6–8) controls thumb IP extension as well as wrist extension. Another physical exam finding to keep in mine is Froment’s sign to asses for ulnar nerve pathology, specifically weakness of the adductor pollicus. The patient is asked to pinch a flat object such as a piece of paper between the thumb and index finger. The examiner then tries to pull the object from their grasp. If there is weakness of the adductor pollicus, the FPL muscle which is innervated by the median nerve is substituted for the adductor pollicus which will cause the IP joint to go into a flexed position. 7. B [6] The dorsum of the fingertips of digits 2 and 3 are innervated by the median nerve. However, the radial nerve provides sensation of these digits until the distal phalanx. It should be remembered that there is considerable overlap in sensory dermatomes and the sensory exam, similar to all exam maneuvers, need to be evaluated in the context of the patient’s presentation as a whole.

5  Physical Examination and Assessment

8. C [7] A. Pain mapping is unreliable due to variable pattern of referred pain. Often patients can describe localized pain; however, it is not uncommon to have pain radiating into the buttocks or even upper hamstrings. B. Gillett Test has been used frequently as a screen for dysfunction but does poor reliability and sensitivity C. Sacroiliac joint dysfunction can prove a diagnostic challenge for clinicians. In a landmark paper in 2007, Laslett described sensitivity and specificity increased to 88% and 78%, respectively, with 3 of 4 provocative maneuvers being positive (thigh thrust, distraction, sacral thrust, and compression) D. Centralization was shown to have a sensitivity of 9% with specificity of 79% in a study by Young et al. 9. D While this individual does exhibit symptoms classically representative of spinal stenosis with neurogenic claudication, he also has upper extremity hyperreflexia (relative to the lower extremities) and a positive Hoffman’s. This warrants further exploration of the cervical spine. Further, a trial of conservative therapy with medications and/or injections is prudent prior to surgical referral in individuals without red flag symptoms. 10. B [8] Chvostek sign, clinical sign of nerve hyperexcitability (tetany) seen in hypocalcemia, is an abnormal reaction to the stimulation of the facial nerve. When the facial nerve is tapped in front of tragus the facial muscles on the same side of the face will contract momentarily. 11. B [9] The elevated arm test of Roos (90° abduction and external rotation of the shoulder while opening and closing the hands for 3  min) seems to have diagnostic value. Adson’s maneuver has questionable ­diagnostic value. Tinel’s and Phalen’s tests

References

are more appropriate for carpal tunnel syndrome. Spurling’s test is used to reproduce radicular pain by foraminal compression. 1 2. C [10, 11] Pain on walking and relieved on sitting, pain relief on using a cane in opposite hand, Trendelenburg gait, pain limited hip ROM, pain provocation on external hip rotation, and tenderness in the region of the hip joint indicate hip joint pain (c). Lumbar facet joint pain is localized in the low back and may be referred to buttocks and thigh, is worse on standing and extension movements, limits standing and walking activities; lumbar extension and facet loading procedures provoke pain, tenderness is localized to the facet areas (paraspinous); unless there is coexisting hip joint pathology, hip joint is non-­ tender and has normal ROM (a). The sacroiliac joint can cause pain on sitting or standing or both, is located in the very low back (distal to the lumbosacral junction) and referred into the buttock, groin and thigh, does not limit lumbar or hip ROM in absence of coexisting pathology, is accompanied by tenderness overlying the PSIS, in the buttock, and along the sacral margin; positive tests include FABERS test (combined flexion abduction and external rotation of the hip provokes sacroiliac pain), Gaenslen’s sign (pain in sacroiliac area on dropping unsupported leg over the edge of the table, while opposite leg remains flexed) and pelvic asymmetry (b). Piriformis can cause pain by (i) compressing the sciatic nerve when it has an aberrant course through the muscle, resulting in local and referred pain and abnormal neurological symptoms in posterior thigh and calf (pseudo radicular); (ii) a second mechanism of pain is due to chronic muscle shortening, called piriformis strain, and presents as deep buttock pain aggravated by sitting, climbing stairs and squats. On examination there is tenderness either in the belly of the piriformis or more distally near its insertion into the greater trochanter, passive internal

31

hip rotation is reduced and resisted abduction with the hip adducted and flexed may reproduce pain over the piriformis. Pain may also be reproduced by resisted external rotation with the hip and knee flexed, beginning from position of internal rotation, so that end range is tested (d). In trochanteric bursitis, pain is localized to the area of the greater trochanter on the lateral aspect of the hip and may be referred along the lateral aspect of the thigh. It is aggravated by hip movements such as climbing stairs or getting out of the car. The site of tenderness is immediately above the greater trochanter (for the gluteus medius bursitis) or over the later aspect of the greater trochanter (e). 1 3. E [12] Pain is entirely a subjective experience. As a result, personal factors impact the process and accuracy of pain assessment. Cultural, ethnic, and linguistic patterns may influence both how the pain is expressed and how it is interpreted. Assessment should include a biopsychosocial model, in which the complaint of pain is viewed in a more global framework. The purpose of a psychological evaluation of a patient with chronic pain is to evaluate the impact of the pain on the patient’s functioning and to determine the role that the patient’s psychological makeup has on his or her experience of pain.

References 1. Scholten P, Chekka K, Benzon HT. Physical examination of the patient with pain. In: Benzon HT, editor. Essentials of pain medicine. Elsevier; 2018. 2. Harb A, Kishner S.  Modified Ashworth scale. [Updated 31 May 2020]. In: StatPearls [Internet]. Treasure Island, FL: StatPearls Publishing; 2020. https://www.ncbi.nlm.nih.gov/books/NBK554572/. 3. Shahrokhi M, Asuncion RMD.  Neurologic exam. [Updated 6 May 2020]. In: StatPearls [Internet]. Treasure Island, FL: StatPearls Publishing; 2020. https://www.ncbi.nlm.nih.gov/books/NBK557589/. 4. Dydyk AM, Das JM.  Radicular back pain. [Updated 14 Apr 2020]. In: StatPearls [Internet]. Treasure Island, FL: StatPearls Publishing; 2020. https://www. ncbi.nlm.nih.gov/books/NBK546593/.

32 5. Konin JG, et  al. Special tests for orthopedic examination. 3rd ed. Thorofare, NJ: SLACK Incorporated; 2006. 6. Whitman PA, Adigun OO.  Anatomy, skin, dermatomes. [Updated 14 Sep 2020]. In: StatPearls [Internet]. Treasure Island, FL: StatPearls Publishing; 2020. https://www.ncbi.nlm.nih.gov/books/NBK535401/. 7. Laslett M.  Evidence-based diagnosis and treatment of the painful sacroiliac joint. J Man Manip Ther. 2008;16(3):142–52. https://doi.org/10.1179/ jmt.2008.16.3.142. 8. Jesus JE, Landry A.  Images in clinical medicine. Chvostek’s and Trousseau’s signs. N Engl J

5  Physical Examination and Assessment Med. 2012;367(11):e15. https://doi.org/10.1056/ NEJMicm1110569. 9. Warfield C. Principles and practice of pain medicine. New York: McGraw-Hill; 2004. p. 327–8. 10. Hoppenfled S.  Physical examination of the spine and extremities. Norwalk: Appleton & Lange; 1976. p. 143–69, 237–61. 11. Brukner P, Khan K. Clinical sports medicine. Sydney: McGraw-Hill; 1997. p. 265–315. 12. Edwards CL, Fillinghim RB, Keefe F. Race, ethnicity and pain. Pain. 2001;94(2):133–7.

Part II Psychology

6

Psychological Evaluation

Questions

3. Which of the following patients does NOT exhibit a factor shown to increase suicidal 1. Which of the following components should risk in chronic pain patients be evaluated during psychological clinical A. 74-year-old female with a past medical evaluation of patients in the pain clinic? history of thyroid disorder, weight gain, A. Sleep patterns and diabetes mellitus type 2. B. Pain-related beliefs B. 37-year-old male with chronic low back C. Cognition pain for 34 years with pain range of 8–10 D. Pain-related behaviors on a 0–10 point scale E. All of the above C. 23-year-old female with fibromyalgia 2. A 38-year-old gentleman with persistent who expresses lying in bed for 3 h each radicular pain following an L5–S1 micro-­ night prior to falling asleep diskectomy procedure 14  months ago is D. 23-year-old female with fibromyalgia being evaluated for a spinal cord stimulator who states her pain ranges from 10–1000 trial. Which of the following notes from the on a 0–10 point scale psychological evaluation is a valid reason to E. All of the above have increased risk for postpone the trial in favor of further psychosuicide in chronic pain patients logical treatment and/or education 4. A 24-year-old female with acute L5 radicu 1. Patient believes they will have no pain or lar pain after lifting a 74 kg weight while liftdiscomfort following the implantation ing at the local gym. She rates her pain an 8 2. Patient has mild depression treated with on a 0–10-point scale and is clearly uncompsychotherapy fortable. You set her up for an L5 transfo 3. Patient has a history of several intoxicaraminal epidural steroid injection. She asks tion arrests including 1 month prior for “something to help with pain.” She also 4 . Patient has seen three other providers states that her primary care provider and the over the last month and gotten short supurgent care providers gave her enough oxyplies of opioids from each in a different codone for 1 tab at night. However, she feels hospital network that she needs “something better. One a day A. 1, 3, 4 just isn’t cutting it!” How would you describe B. 1, 2, 3 her behavior? C. 3, 4 A. Drug-seeking behavior D. All of the above B. Addiction © The Author(s), under exclusive license to Springer Nature Switzerland AG 2021 M. Suer, N. Sehgal, Questions and Answers in Pain Medicine, https://doi.org/10.1007/978-3-030-68204-0_6

35

36

C. Pseudo-addiction D. Malingering 5. A 47-year-old female is being evaluated in the cancer pain clinic. You determine she is a candidate for chronic opioid therapy and administer the Screener and Opiate Assessment of Patients with Pain-Revised (SOAPP-­R). Which of the following items is TRUE regarding this questionnaire? A. It is validated for all patients with pain who are being evaluated for opioid therapy B. It is only valid for patients with chronic cancer pain C. A score less than 18 indicates there are no issues with prescribing opioids D. It was developed based on expert consensus and takes less than 10  min to complete 6. A 47-year-old man is referred to the pain clinic for treatment recommendations for fibromyalgia. Throughout the encounter, his wife is seen rubbing his back and notes that she “does everything” for him because of his pain. She stresses that he is unable to do household chores and that she encourages him to rest while she does many of the tasks around the home. This is an example of: A. Classical conditioning B. Operant conditioning C. Munchausen by proxy D. Habituation 7. You are evaluating a 31-year-old female for multiple complaints that have been ongoing for 2 years. She has multiple pain complaints including abdominal pain, back pain, right leg pain, fatigue, and occasional shortness of breath. She finds that she spends many hours per day worrying over her health. She has been evaluated by multiple medical providers who have been unable to identify an underlying etiology for her symptoms. What is true of her underlying diagnosis A. Symptoms have to have started prior to age 30 B. There is an equal distribution amongst males and females

6  Psychological Evaluation

C. Psychotherapy is the recommended first treatment of choice D. One must have pain in four different sites, two gastrointestinal complaints, one sexual symptom, and one pseudo-­ neurological symptom to meet diagnostic criteria 8. A 24-year-old female is evaluated in the pain clinic for somatic symptom disorder. Her abdominal pain has been present for 8 months. She also believes there is “something seriously wrong with me” despite being reassured by multiple providers that there is no organic pathology to account for her symptoms. Despite noting her pain to be quite severe, she has continued working as a corporate lawyer during this time. She scores 5 on GAD-7. How would you classify her disorder? A. Persistent, mild B. Non-persistent, moderate C. Persistent, severe D. Persistent, moderate 9. Which of the following is NOT considered a diagnostic criteria of functional neurological symptom disorder? A. The patient has at least one symptom of altered voluntary motor or sensory function B. Clinical findings provide evidence of incompatibility between the symptom and recognized neurological or medical conditions C. The symptom or deficit is not better explained by another medical or mental disorder D. The symptoms are not intentionally produced or feigned 10. A 23-year-old female presents to the ED for sudden-onset right arm weakness. CT and MRI of the head do not show underlying pathology and NCS shows no abnormalities. EMG demonstrates normal insertional activity with no activity seen on activation of any muscles in the upper extremities. Which of the following should be involved in her treatment plan? A. Psychological evaluation B. Physical Therapy

Questions

C. Occupational Therapy D. Caregiver education E. All of the above 11. A 43-year-old overweight woman is evaluated in the pain clinic for chronic abdominal pain of 8  years duration. On physical exam, you notice several surgical scars across her abdomen to which she responds she has had an appendectomy, cholecystectomy, exploratory laparotomy, and lysis of adhesions without relief of her symptoms. She notes persistent right upper quadrant abdominal pain without exacerbating factors including eating. She has diffuse tenderness to palpation throughout her abdomen and normal bowel sounds. She has no pain with straight leg raise against resistance. When you mention interventional procedures for pain management, she quickly explains which injection she would like you to perform. Her presentation is most consistent with: A. Factitious disorder B. Malingering C. Undiagnosed organic illness D. Fibromyalgia 12. A 57-year-old male is seen in the clinic for evaluation of acute onset low back pain. He states the pain began while at work when he was lifting a heavy keg onto a table. He reports stabbing pain in the right side of his low back radiating down the lateral aspect of the leg. He has no weakness or sensory changes in the leg. When he is changing clothes prior to the physical exam, his wife tells you that he actually hurt his back the day before when he was pulling weeds in their garden but is about to be fired from work and needs to be put on disability. Lumbar MRI reveals right L4–5 paracentral disc bulge. This is best characterized as: A. Malingering B. Partial malingering C. False imputation D. Factitious disorder 13. You perform the following exam maneuvers on an individual suspected of malingering.

37

Which of the following pairs correctly matches the test with a positive test for malingering A. Hoover Sign—In supine, you place both hands under the patients heels. The patient is unable to raise their paretic limb. On raising the unaffected limb, you feel pressure on the hand of the paretic limb. B. Waddell Test—Positive straight leg raise with negative seated straight leg raise C. Mankopf—Patient stands on one leg and flexes the symptomatic leg and raises it to the chest. The patient reports pain with doing so and is unable to complete it D. Arm drop—You hold the patients had above their face while they are in the supine position and release their hand. It directly goes and hits their nose causing significant pain. 14. A patient in a waiting room observes another patient who complains loudly of severe pain and receives solicitous comments from his spouse. Later that day, the first patient is observed making similar complaints to his own spouse. This is an example of: A. Classical conditioning B. Operant conditioning C. Social learning theory D. Personality disorder E. Plagiarism 15. The incidence of depression in chronic pain patients is most commonly reported to be between: A. 5 and 15% B. 75 and 85% C. 30 and 60% D. 20 and 40% 16. Which of the following is not recommended when treating a chronic pain patient with borderline personality disorder? A. Confrontation of problem behaviors B. Consistent limit setting C. Benevolent and accommodating attitude D. Explicit treatment contracts E. Communication between different treating professionals

38

17. Which of the following psychosocial variable is the least likely explanation for intercultural differences in coping with ­ chronic pain? A. Locus of control B. Belief in mind-body dualism C. Frequency of depression D. Level of education 18. The cognitive-behavioral approach uses: A. Collaborative, time-limited, educational, and problem-oriented techniques B. Long-term, frequent sessions, with emphasis on the unconscious mind C. Reinforcement contingencies as the primary influence on behavior D. Emphasis on achieving insight into the causes of mood and behavior E. Frequent repetition of patient’s comments by therapists 19. Overall goals for psychological treatment include: A. Reduce pain and pain related disability B. Treat co-morbid psychiatric illness, e.g. depression C. Increase perception of control and self-efficacy D. Address impact of pain on family and marital function E. All of the above 20. Which psychological treatment primarily focuses on decreasing learned pain behavior and replacing maladaptive responses with adaptive behaviors inconsistent with the sick role? A. Relaxation training B. Operant interventions C. Biofeedback D. Self-hypnosis E. Psychoanalysis 21. After hip replacement a patient is prescribed short-acting opioid analgesics to be given orally every 4 h as needed. After 2 days the patient’s regimen is then changed to offer the same dose every 4  h on a scheduled basis. What behavioral technique has been used? A. Removal of contingency between pain behavior and reinforcement B. Punishment

6  Psychological Evaluation

C. Positive reinforcement D. Variable-interval reinforcement E. Aversive conditioning 22. Alternately contracting and relaxing specific muscle groups, diaphragmatic breathing and guided imagery are features of: A. Operant pain programs B. Relaxation training C. Biofeedback D. Hypnosis E. Cognitive behavioral therapy 23. Increasing awareness of a physiologic process of which an individual is typically unaware and using this information to control this involuntary process is the underlying principle of: A. Operant pain programs B. Relaxation C. Biofeedback D. Hypnosis E. Cognitive behavioral therapy 24. Which of these is a maladaptive pain coping skill? A. Behavioral distraction B. Relaxation C. Pacing of activities D. Pain-contingent rest E. Positive self-talk 25. A psychological intervention that may require patients to monitor their thoughts about pain or a pain-related situation, identify negative thoughts, and generate accurate, adaptive thoughts to replace the negative thoughts is called as: A. Operant pain programs B. Relaxation C. Biofeedback D. Hypnosis E. Cognitive behavioral therapy 26. A patient who consciously fakes pain or another medical condition in order to assume the “sick role” without any obvious external reward or gain is most likely: A. Suffering with a somatoform disorder as the result of unresolved psychological B. conflicts from the past C. To meet the criteria for a factitious disorder

Answers

. Believed to be malingering D E. Have a personality disorder 27. Which psychological disorder has primary gain, meaning that the individual is intentionally faking in order to obtain an external reward, such as money from a court case. A. Factitious disorder B. Malingering C. Conversion disorder D. Somatoform disorder E. All of the above 28. What essential feature do pain-related somatoform disorders share? A. Patient’s symptoms resolve once medical reassurance is offered B. Patient spends considerable time, effort and money looking for the cure C. Patient maintains high level of functioning with symptoms D. Patient has little distress despite symptoms 29. In managing patients with a somatoform disorders, which statement is not recommended? A. Confront patients with conversion disorders. B. Provide reassurance that neurological functioning will improve with conversion disorder patients. C. Educate patients with somatization disorder about on how psychological conflicts impact the physical stress response. D. Teach patients relaxation skills to lower arousal in the body.

Answers 1. E [1] Important assessment domains include pain-related disability and behavior, psychological function, sleep, pain-related beliefs, and behavior 2. A [2] Exclusion criteria for neuromodulation include major psychiatric disorder including: poor compliance and/or insufficient understanding of the therapy, lack of appropriate

39

social support, drug and alcohol abuse, and drug-seeking behavior. 3. A [3] Factors shown to have increased risk of suicide in the chronic pain population include comorbid depression, high pain intensity and duration, presence of sleep-­onset insomnia, high catastrophizing, and feelings of hopelessness. The individual in A does not exhibit any of these factors 4. C [4, 5] Physicians can, at times, be fearful of prescribing appropriate pain medications in fear of fostering addiction. However, this often results in under-treatment of pain. Patients who have poorly-treated pain often seek pain relief from other providers. However, such behavior is deemed pseudo-addiction as it is not true signal of opiate misuse as much as it is inadequate pain relief. 5. D [6] A. It is validated for patients with chronic pain whether they have cancer or not. It is not valid for patients with acute pain B. While it can be used for patients with cancer, it has been validated for patients with chronic pain C. While 18 is the cutoff score, the test is NOT a lie detector and patients may mis-­ represent the true answers on the questionnaire. D. The Screener and Opiate Assessment of Patients with Pain—Revised (SOAPP-R) is a 24-item self-reported questionnaire designed to assess the risk for drug abuse and aberrant medication-related behavior. 6. B [7] A. Classical conditioning—This is a behavioral conditioning model in which a stimulus produces a reflexive behavior. The classic example, Pavlov’s dog, demonstrates that a stimulus (ringing of bell) produces a behavior (salivation) when the stimulus is associated/learned with the behavior B. Operant conditioning—In operant conditioning, stimuli that are present when a

40

behavior is performed, come to control that behavior. In contrast to classical ­conditioning, it is not reflexive. Rather, it is voluntary. C. Munchausen by proxy is a factitious disorder wherein the caregiver makes a dependent person feign mental or physical illness in order to gain attention D. Habituation is a form of learning in which one decreases its response to a stimulus after repeated/prolonged presentations 7. C [8, 9] Somatoform disorder has been removed from the most recent DSM-5 and replaced with somatic symptom disorder. Accordingly, the diagnostic criteria have changed and no longer include a focus on the negative symptoms that were previously used. However, psychological treatment remains the treatment of choice with CBT showing good results in clinical trials. Answers A and D represent the former somatoform disorder diagnostic criteria. There is a higher incidence of somatic symptom disorder in females compared to males. 8. A [10] Diagnostic criteria for somatic symptom disorder include modifiers to specify if the patient has predominantly pain, persistent symptoms, and to classify severity (mild, moderate, or severe). For mild severity, one should display only one of the symptoms specified in Criterion B.  Moderate severity has two or more of the Criterion B symptoms fulfilled. Severe adds multiple somatic complaints (or one very severe somatic symptom). The individual in this case fits the mild severity as she has disproportionate and persistent thoughts about her symptoms; however, she does not meet GAD-7 criteria of anxiety disorder (greater than 8) or spend excessive time and energy devoted to the symptoms as she is able to continue work as a corporate lawyer. 9. D [11] While the former DSM-IV criteria included D, this is no longer included in the DSM-V criteria for diagnosing conversion

6  Psychological Evaluation

disorder. The diagnostic criteria of conversion disorder are A, B, C and the symptom deficit causes clinically significant distress or impairment in social, occupational, or other important areas of functioning or warrants medical evaluation. 10. E [12] For treatment of conversion disorder (functional neurological symptom disorder), education of the condition is the first-line treatment although this is rarely enough to treat. You should utilize physical therapy for motor symptoms and include cognitive based therapy for psychological symptoms. The most commonly used medications for treatment are anti-depressants. The most important aspect of treating is to utilize a multidisciplinary approach. 1 1. A [11] A. Factitious disorder is falsification of physical or psychological signs/symptoms, or induction of injury or disease, associated with identified deception. The individual presents themselves as ill (compared to factitious disorder imposed on others; Munchausen by proxy). Further, these individuals are often very well versed in textbook presentation of illness and are eager to undergo invasive procedures. B. In comparison to factitious disorder, patients who are malingering have similar presentation but are looking for personal gain by obtaining illness/treatment C. Organic illness in this individual is unlikely given the multiple procedures she has had performed including exploratory laparotomy. D. She does not meet the diagnostic criteria of fibromyalgia 1 2. C [11] A. Malingering—Malingering involves patients fabricating symptoms that do not exist at all for personal gain. This individual likely has real symptoms given the mechanism and pattern of pain. He has evidence of L5 radicular pain and evidence on MRI of likely L5 nerve impingement making this less likely.

Answers

B. Partial malingering involves exaggeration of symptoms that do exist. While this is possible in this individual, there is no frank evidence that he is exaggerating his symptoms C. False imputation is the best answer. It is clear that he is trying to avoid losing his job and they believe a work-related injury will require you to give work restrictions and allow him to keep his job. D. He does not meet the diagnostic criteria of factitious disorder as he does have personal gain as the primary motive for his actions. 1 3. A [13, 14] A. Correct. Hoover Sign—In supine, you place both hands under the patients heels. The patient is unable to raise their paretic limb. On raising the unaffected limb, you feel pressure on the hand of the paretic limb. B. Waddell Test requires positive signs from three or more categories including tenderness, stimulation, distraction, regional disturbances, and overreaction. 1. Tenderness tests: superficial and diffuse tenderness and/or nonanatomic tenderness 2. Simulation tests: these are based on movements which produce pain, without actually causing that movement, such as axial loading and pain on simulated rotation 3. Distraction tests: positive tests are rechecked when the patient’s attention is distracted, such as a straight leg raise test 4. Regional disturbances: regional weakness or sensory changes which deviate from accepted neuroanatomy 5. Overreaction: subjective signs regarding the patient’s demeanor and reaction to testing C. Mankopf—1700  g pressure applied to the middle phalanx of the second finger of the nondominant hand. True pain should increase heart rate D. Arm drop—Hold paretic hand above face and drop it. If hand misses face, paresis is nonorganic.

41

14. C [15] The patient who observes, and then duplicates, a particular sort of pain behavior is demonstrating the principles of social learning theory. When observed, the behavior appeared to achieve a desirable result (solicitous attention from spouse), making it an attractive candidate for imitation. Cognitive and environmental factors act together to drive social learning. Classical conditioning describes learning by repeatedly pairing a particular stimulus with a neutral stimulus, leading to the elicitation of behavior using the neutral stimulus alone. Operant conditioning describes learning created by reinforcing particular behaviors to increase their frequency and strength. A personality disorder is a pervasive pattern of experience and behavior that is abnormal with respect to any two of the following: thinking, mood, personal relations, and the control of impulses. 15. C [15] Depression is very common in patients with chronic pain. Most studies report prevalence rates between 30 and 60%. The exact rate cited depends upon the type of pain, condition investigated, the diagnostic criteria, the assessment tools, and the population studied. 16. A [16] All but c are correct elements of treatment for the patient with borderline personality disorder (BPD), which involves a pattern of unstable interpersonal relationships, self-­ image, and affect, with marked impulsivity. Patients with BPD can cause chaos in the clinic if inappropriately managed. Clear and explicit limit-setting and well-defined goals are essential. Excessive accommodation or flexibility in the set goals will facilitate the patient’s emotional instability and lead to treatment failure. 17. C [17–19] Locus of control, educational level, and belief in mind-body dualism (or lack thereof) have all been shown to impact the efficacy of coping and the success of health care interventions as measured in multicultural groups.

42

The frequency of depression does not satisfactorily explain cross-cultural effects. 18. A [20] Cognitive-behavioral psychotherapy uses education, collaboration, and a problem-­ focused approach to identify behaviors that contribute to stress and dysfunctional mood, plan practical modifications to those behaviors, and implement the modifications. The key concept is that a patient’s own thoughts, not the thoughts or actions of others, determine his/her behavior. Therapy is usually time-limited and ends when the specific goals of therapy are accomplished. The other answer options describe other psychotherapies. Choice b describes psychoanalysis; choice c describes strict behavioral therapy; and choice d describes insight-­ oriented psychotherapy. Choice a describes client-centered (Rogerian) psychotherapy. 1 9. E [1] Psychological interventions have been empirically demonstrated to reduce pain and suffering in patients with a wide variety of chronic pain states. Subpopulations that can benefit from referral include patients with depression or anxiety, those demonstrating excessive disability, those with negative affect, family/social factors or “secondary gain”, and problematic medication use. 2 0. B [1] In the operant model of pain, the primary focus of intervention is the behavior of the patient. Reinforcers increase and punishments decrease the likelihood of a behavior occurring. Ideally the environment must allow for an opportunity to control the social consequences of pain behavior and shape new “wellness” behavior. Pacing and activity modulation are important components of an operant behavioral program. Desirable behavior is “shaped” by gradually increasing its nature, frequency or duration, with the aim of increasing the behavior while managing the consequences, which includes removing any punishment (pain) and introducing reinforcements (e.g. social attention).

6  Psychological Evaluation

21. A [1] Changing from as-needed to fixed-­ interval prescribing removes the contingency between the patient’s report of pain (pain behavior) and pain relief (the reinforcer). The other choices refer to different aspects of conditioning theory, none applicable here. 22. B [1] Relaxation training has two components: (1) repetitive focus on a word, body sensation or muscle activity; and (2) adopting a passive attitude towards thoughts unrelated to one’s attentional focus. Common techniques used to teach relaxation include systematically tensing and relaxing specific muscle groups, diaphragmatic breathing and guided imagery. Stress or pain leads to subtle increases in muscle tension, which can exacerbate pain. Relaxation training attempts to break the pain-muscle tension-pain cycle. There is evidence supporting use of relaxation for pain management. 23. C [21] Biofeedback for pain management usually entails providing feedback about muscle tension, typically using surface electromyography at painful muscles. Biofeedback is useful in treating Raynaud’s phenomenon, tension and migraine headaches, and low back pain. 24. D [21] All others are examples of adaptive pain coping. The primary goal of teaching pain coping skills is to increase perception of pain as a controllable experience and decrease the use of maladaptive coping strategies. Automatic rest when pain occurs can lead to deconditioning and impaired function. 25. E [21] Cognitive behavioral interventions include components of (1) behavioral model, especially relaxation training and some components of operant conditioning (2) cognitive restructuring, such as attitudes and beliefs that underlie maladaptive emotional and behavioral responses. Cognitive restructuring

References

focuses on role of cognitive factors such as attitudes, thoughts, and beliefs in determining emotional and behavioral responses to pain. These interventions challenge negative selftalk, catastrophizing and replace these selfstatements with more positive statements that reduce negative affect, emphasize control and encourage adaptive coping. 26. C [22] In Munchausen syndrome or Factitious Disorder the patient consciously assumes the sick role and may even subject himself/herself to invasive surgical procedures with no obvious external rewards. The malingerer consciously fakes as well, but has an external reward that he is trying to obtain and most likely would not agree to invasive medical procedures. 2 7. B [22] The difference between malingering and factitious disorder, conversion disorder and other somatoform disorders lies in motivation. Primary gain is the unconscious production of physical symptoms to ward of awareness of anxiety and other psychological conflict. It is commonly seen in somatoform disorders such as conversion and somatization disorder. Malingers are aware of faking due to a desire to obtain an external reward such as money. The malingerer is motivated by external factors rather than an in a conscious need to maintain the sick role. Because of the conscious and intentional nature of the behaviors, malingering is not considered a psychiatric disorder. Factitious disorder intentionally produces or feigns physical or psychological signs or symptoms but the motivation for the behavior is to assume the sick role. 2 8. B [23, 24] In all pain-related somatoform disorders, pain is the predominant feature and causes distress sufficient to interfere with daily functioning and warrants seeking medical attention. Additionally, the patient usually spends considerable time, effort and money looking for the physician who will have the “cure,” despite the report of negative or mini-

43

mal findings, failed intervention or repeated declarations that nothing can be done to resolve the underlying problem. 2 9. A [23] Do not confront conversion disorder patients; however, do provide reassurance that function will improve and that the condition is not life threatening. Somatizing patients do not recognize the effect of their emotions. Although the onset of the symptoms can occur during a stress period, this connection tends to be denied or forgotten by the patient. The symptoms mask the underlying conflict which may allow the patient to ignore or not have to deal with it.

References 1. Leerman SF, Haythornthwaite J. Psychological evaluation and testing. In: Benzon HT, editor. Essentials of pain medicine. Elsevier; 2018. 2. Beltrutti D, Lamberto A, Barolat G, et  al. The psychological assessment of candidates for spinal cord stimulation for chronic pain management. Pain Pract. 2004;4:204–21. 3. Tang NK, Crane C.  Suicidality in chronic pain: a review of the prevalence, risk factors and psychological links. Psychol Med. 2006;36:575–86. 4. Linares AD.  Opioid pseudoaddiction: a casualty of the war on drugs, racism, sexism, and opiophobia. Quinnipiac Health Law J. 2011;15:89–125. 5. Federation of State Medical Boards of the United States Inc. Resources for pharmacovigilance and pain management; 2010. 6. Butler SF, Fernandez K, Benoit C, Budman SH, Jamison RN.  Validation of the revised Screener and Opioid Assessment for Patients with Pain (SOAPP-R). J Pain. 2008;9:360–72. 7. Skinner BF.  The behavior of organisms: an experimental analysis. New York: Appleton-Century-Crofts; 1938. 8. Kellner R.  Functional somatic symptoms and hypochondriasis. Arch Gen Psychiatry. 1985;42: 821–33. 9. Rief W, Mewes R, Martin A, Glaesmer H, Brähler E. Evaluating new proposals for the psychiatric classification of patients with multiple somatic symptoms. Psychosom Med. 2011;73(9):760–8. 10. Substance Abuse and Mental Health Services Administration. Impact of the DSM-IV to DSM-5 changes on the national survey on drug use and health [Internet]. Rockville, MD: Substance Abuse and Mental Health Services Administration (US); 2016. Table  3.31, DSM-IV to DSM-5 somatic symptom

44 disorder comparison. https://www.ncbi.nlm.nih.gov/ books/NBK519704/table/ch3.t31/. 11. American Psychiatric Association. Diagnostic and statistical manual of mental disorders. 5th ed. Washington, DC: American Psychiatric Association; 2013. 12. Stone J, Sharp M. Conversion disorder in adults. In: Post TW, editor. Uptodate. Waltham, MA; 2020. 13. D’Souza RS, Dowling TJ, Law L.  Waddell sign. [Updated 10 Jul 2020]. In: StatPearls [Internet]. Treasure Island, FL: StatPearls Publishing; 2020. https://www.ncbi.nlm.nih.gov/books/NBK519492/. 14. Koehler PJ, Okun MS.  Important observations prior to the description of the Hoover sign. Neurology. 2004;63(9):1693–7. https://doi.org/10.1212/01. wnl.0000142977.21104.94. PMID: 15534257. 15. Koestler AJ, Doleys DM. The psychology of pain. In: Tollison CD, et  al., editors. Practical pain management. 3rd ed. Philadelphia: Lippincott Williams & Wilkins; 2002. p. 26–39. 16. Saper JR.  Borderline personality disorder and the chronic headache patient: review and management recommendations. Headache. 2002;42(7): 663–74. 17. Cano A, Mayo A, Ventimiglia M. Coping, pain severity, interference, and disability: the potential medi-

6  Psychological Evaluation ating and moderating roles of race and education. J Pain. 2006;7(7):459–68. 18. Bates MS, Rankin-Hill L, Sanchez-Ayendez M. The effects of the cultural context of health care on treatment of and response to chronic pain and illness. Soc Sci Med. 1997;45(9):1433–47. 19. Bates MS, Rankin-Hill L. Control, culture and chronic pain. Soc Sci Med. 1994;39(5):629–45. 20. Mayo Clinic. Psychotherapy: an overview of the different types of therapy. 2006. http://www.mayoclinic. com/health/psychotherapy/MH00009. 21. Haythornthwaite JA, Heinberg LJ.  Psychological intervention in chronic pain. In: Benzon HT, Raja SN, Molloy R, Liu S, Fishman S, editors. Essentials of pain medicine and regional anesthesia. 2nd ed. Philadelphia, PA: Elsevier/Churchill Livingstone; 2005. 22. Craig KD, Badali MA.  Introduction to the special series on pain deception and malingering. Clin J Pain. 2004;20:377–82. 23. Argoff CE, McCleane G.  Pain management secrets. 3rd ed. Philadelphia, PA: Mosby Elsevier; 2009. 24. Fishbain DA, Cutler RB, Rosomoff RS, Rosomoff HL.  The problem-oriented psychiatric examination of the chronic pain patient and its application to the litigation consultation. Clin J Pain. 1994;10:28–51.

7

Psychological Interventions and Pharmacology

Questions 1. What percentage of patients with chronic pain have co-morbid major depression A. 10–30% B. 30–50% C. 50–70% D. >70% 2. Which of the following is true regarding pain and depression? A. Treating underlying psychiatric illness will result in improvement in psychiatric illness results in diminished pain levels, improved functionality and quality of life. However, acceptance of chronicity of pain will only come with CBT B. Major depression, though not low levels of depression, will worsen the physical impairment associated with chronic pain C. Depressive symptoms often present with the Beck triad which is hopeless, hapless, and helpless D. Untreated or undertreated major depression has a lifetime risk of death through completion of suicide of 5–10%. 3. A 32-year-old female with depression, fibromyalgia, chronic migraines, and IBS is seen in the ED following onset of agitation and increased headache. Vitals read 38.4 °C, BP 145/89, RR 18, O2 saturation 98% on room air. She is being treated with duloxetine for fibromyalgia and has recently started alpha

lipoic acid. In the headache clinic, she was recently started on Botox injections in addition to propranolol for migraine prophylaxis. She also takes sumatriptan for migraine treatment. Which of the following medications likely contributed to her current situation? A. Botox B. Propranolol C. Sumatriptan D. Alpha Lipoic Acid 4. Which of the following medications has the fewest anti-cholinergic side effects? A. Amitriptyline B. Nortriptyline C. Imipramine D. Desipramine 5. You are evaluating a 49-year-old male with 30-year history of type II diabetes mellitus. You diagnose him with fibromyalgia and believe he has depressive symptoms as well. After deciding to start him on a tricyclic anti-­ depressant, which of the following should be checked prior to initiating treatment? 1. Baseline EKG 2. Liver function tests 3. BUN and Creatinine 4. Electrolytes A. 1, 2, 3 B. 1, 3, 4 C. 2, 3, 4 D. All of the above E. None of the above

© The Author(s), under exclusive license to Springer Nature Switzerland AG 2021 M. Suer, N. Sehgal, Questions and Answers in Pain Medicine, https://doi.org/10.1007/978-3-030-68204-0_7

45

46

7  Psychological Interventions and Pharmacology

6. A 78-year-old female is started on Cymbalta B. Refer her to physical therapy for treatment of fibromyalgia. Which of the C. Reassure the patient that there is nothing following is the recommended starting doswrong with her and that her anxiety is age for this patient? simply getting the best of her at this A. 30 mg taken each night point B. 60 mg taken each morning D. Refer her to psychotherapy C. 20 mg taken each night 1 1. Which of the following statements regarding D. 30 mg taken each morning anxiety and antidepressant medications is 7. Which of the following is NOT a common true? side effect seen with duloxetine? A. Antidepressants can be used in diminish A. Nausea ing the overall level of anxiety, prevent B. Fatigue ing panic attacks, and treating acute C. Insomnia anxiety. D. Agitation B. Although often used to treat depression 8. Which of the following medications is FDA-­ with anxious features, bupropion can approved for the treatment of fibromyalgia have stimulating effects, making it a less and depression? attractive choice A. Duloxetine C. When using SSRI’s, dosages are similar B. Gabapentin when treating depression and anxiety C. Milnacipran D. While often used to treat depression and D. Wellbutrin anxiety, duloxetine is not FDA approved 9. A 45-year-old, otherwise healthy, female is for the treatment of generalized anxiety started on a medication for treatment of fibrodisorder myalgia. One month later, she presents to the 12. Following a dental procedure, a 45-year-old hospital later with symptoms concerning for man develops severe, intermittent, lancinatpneumonia. CBC reading includes neutrophils ing pain along the right mid and lower facial 450/μL.  Which of the following medications regions. His pain can be brought on by light was likely started on this individual? touch. What is true regarding the first-line A. Trazodone treatment for this ailment? B. Tramadol A. Depression of the CYP450 system C. Oxycodone B. Often started at 300  mg at night and D. Mirtazapine titrated up to effect 10. A 57-year-old female with chronic pain is C. Topically applied Na-channel blocker evaluated in the pain clinic. She requested D. Requires blood-level monitoring todays appointment as she has had increased 13. A 43-year-old female is evaluated in the ED restlessness and fatigue over the last for status migrainosus. She takes propranolol 7 months. Her family has also told her that and prophylaxis and sumatriptan  ×  2 today she is more irritable though she relates this for acute treatment without relief. Soon after simply to less sleep. Her upper back muscles receiving treatment in the ED, she develops have also been more painful. She fears that temperature 39.2  °C, labile hypertension, her pain is getting worse and has been avoidRR 32, HR 109, and muscle rigidity. Which ing many activities for fear that they will medication was likely given to precipitate increase her pain. What is the correct next her condition? step in management of her symptoms A. IV Magnesium A. Perform trigger point injections into her B. Prochlorperazine upper trapezii to relieve her upper back C. Tramadol pain D. Hydromorphone

Questions

14. A 23-year-old man is evaluated for chronic migraines and is started on topiramate for prophylaxis. Which of the following is NOT included in the mechanism of action of topiramate? A. Inhibition of activity-dependent, voltage-­ gated sodium channels B. Blockade of L-type voltage-gated calcium channels C. Enhancement of action of GABA pathway D. Increased carbonic anhydrase activity 15. You start a 67-year-old gentleman on duloxetine for treatment of chronic, neuropathic pain. After 3 weeks, the patient calls and states they have had no change in their symptoms after 3 weeks on 30 mg/day. They have noticed no ill effects with the medication as well. What is the next step to take in treatment of their pain? A. Taper the medication and initiate another medication for treatment of neuropathic pain B. Continue the medication at the current dosage as it often takes 4–8  weeks to reach peak effect C. Increase the dosage to 60 mg/day D. Continue medication at the current dosage and add a second medication for treatment of neuropathic pain 16. The pioneering work in pain medicine by __________ brought about the idea of pain as having an emotional component A. Avicenna and Descartes B. Melzak and Wall C. Turk D. Fordyce 17. A 56-year-old man with chronic pain secondary to shoulder arthritis frequently misses work as a carpenter. He states his boss is always sympathetic toward his pain and that whenever he shows the slightest bit of pain, his boss has him rest. You wish to use this same type of intervention to correct the behavior. A. Start activity pacing such that activities are stopped prior to having pain B. Once pain begins, have the patient start relaxing muscles progressively to reduce tension on the shoulder to decrease pain

47

C. Have the patient evaluated by PT and work on strengthening of the muscles around the shoulder D. Have him work on coping skills in noting that pain is a controllable experience and he should avoid activities that cause pain 18. The key component of hypnosis is: A. Total mind and body relaxation B. Use of another individual to induce an altered mental state C. Suggestibility D. A state of extreme concentration and introspective thought 19. A 53-year-old man with pain secondary to multiple medical issues is seen in the pain clinic. On asking him his job, he states “Oh, I’m just a bump on a log now.” You find that he has not worked in 9 months and is on disability secondary to his multiple medical issues that have resulted in his pain. While he hopes to get back to work soon, he tells you he just can’t take the pain anymore. “It used to be bearable, but it’s just so bad now. I’m always at a 9 and sometimes, I’m at a 12 out of 10. I don’t think I’ll ever get better. I’d better get used to just sitting on the couch.” You refer him to psychotherapy and request they work on _______. A. Coping Skills B. Cognitive Restructuring C. Relaxation Techniques D. ECT for major depression 20. The objective of ________ is not elimination of difficult feelings; rather, it is to be present with what life brings us and to “move toward valued behavior”. A. Cognitive Behavioral Therapy B. Mindfulness C. Acceptance and Commitment Therapy D. Cognitive Analytic Therapy 21. Select the FALSE statement regarding use of biofeedback in treating headaches: A. Blood-volume-pulse biofeedback has the largest effect B. All forms of biofeedback show medium to large effect sizes C. Biofeedback is superior to placebo

48

D. Tension headaches do not respond to biofeedback E. Biofeedback is useful in treating migraine headaches 22. Hypnosis, when used as a treatment for chronic pain, is: A. Not as effective as physical therapy B. A well standardized and accepted mode of therapy C. Significantly helpful in a variety of chronic pain states D. Not useful in chronic pain due to cancer E. Supported by a large number of well-­ designed trials 23. A self-management technique that involves the tensing and releasing of muscles to help with relaxation is called: A. Guided-imagery Relaxation training B. Progressive Muscle Relaxation C. Passive Muscle Relaxation D. Biofeedback 24. A patient is considering starting an exercise program to help with pain management, physical conditioning and weight loss; however, is unsure whether he really wants to start exercising because he is afraid of increased pain. This patient is likely to be at what Stage of Change according to Prochaska and DiClemente’s Transtheoretical Model of Change? A. Contemplation B. Preparation C. Pre-contemplation D. Action 25. Which psychological treatment primarily focuses on teaching families of pain patients to ignore the patients’ pain behaviors, such as facial grimacing, and to provide encouragement for displays of increased level of functioning and independence? A. Relaxation training B. Operant interventions C. Biofeedback D. Self-hypnosis E. Psychoanalysis

7  Psychological Interventions and Pharmacology

Answers 1. B [1, 2] Studies have shown variable correlation percentages, however a 1997 study showed 30–50% of patient with chronic pain also had depression. Certainly, the number of patients with depressed mood or dysthymic disorder would be greater with some studies showing 60–80% of patients with chronic pain having significant psychiatric pathology. 2. C [2] A. Improvement in psychiatric illness results in diminished pain levels, greater acceptance of the chronicity of pain, improved functionality, and improved quality of life. B. Even low levels of depression (“subclinical depression”), will worsen the physical impairment associated with chronic pain C. Depressive symptoms often present with the Beck triad which is hopeless, hapless, and helpless D. Untreated or undertreated major depression has a lifetime risk of death through completion of suicide of 10–15%. 3. C [3] This individual is likely suffering from serotonin syndrome given her agitation and increased temperature with the history of increased use of sumatriptan in the setting of chronic duloxetine usage. One must be wary of the medications that can cause increase in serotonin resulting in serotonin syndrome. Common pain medications that can precipitate serotonin syndrome include TCA’s, SSRI’s, SNRI’s, opioids (tramadol, oxycodone, hydrocodone), triptans. Herbals to keep in mind include St John’s wort, panax ginseng, nutmeg, and yohimbe. See the citation with this answer for an in-depth review of the topic including a figure of medications commonly associated with serotonin syndrome.

Answers

4. D [4] Of these TCA’s, desipramine has the fewest anti-cholinergic side effects followed by 10. nortriptyline. Imipramine and amitriptyline have the high frequency of anti-cholinergic side effects. 5. D [5] All of the above should be checked prior to initiating TCA’s. While not always done in clinical practice, it is recommended. 6. C While the normal starting dosage is 30 mg taken each night, elderly individuals should be started at a lower dosage to ensure tolerance to side effects (especially anti-­ cholinergic side effects). It is not uncommon to increase individuals to 60 mg daily once safety profile has been established at 30 mg daily. 7. D [6] Common side effects with duloxetine include: nausea, headache, constipation, dry mouth, dizziness, fatigue, insomnia. While agitation can occur, it is less common than the other side effects listed. 8. A [6–9] A. Duloxetine is the only medications FDA-­ approved for both fibromyalgia and depression. The other medications approved for fibromyalgia include pregabalin (Lyrica) and milnacipran (Savella). 11. B. Gabapentin is not used for depression but does have mild anxiolytic properties. C. Milnacipran, while only approved in the US for fibromyalgia carries approval for both fibromyalgia and depression in Europe. D. Bupropion is approved for depression, seasonal affective disorder, and smoking cessation (SR) though off-label it can be used for neuropathic pain. 9. D [10, 11] Mirtazapine carries approximately 0.3% risk of agranulocytosis. Other medications that cause lead to agranulocytosis include: carbam-

49

azepine, valproate, penicillin, naproxen, allopurinol, and clozapine among others. D [12] A. While she may benefit from trigger point injections or other myofascial work of her upper back muscles, this is not the correct first treatment option. Often, upper back muscles are tight due to stress, increased pain, or another underlying disorder and these should be treated prior to simply treating the pain complaint in this area. B. Similarly, PT would work well for her myofascial pain and she may benefit further from PT given her fatigue. However, treating the underlying anxiety is more important as this can frequently cause fatigue. C. While often reassuring the patient is beneficial, the tone of this answer is condescending and one should not simply tell patients that their symptoms are nothing to worry about. Further, her symptoms have been present for 7  months at this point and need more direct intervention D. Correct—psychotherapy, specifically CBT, is the treatment of choice in anxiety disorder. While she may benefit from medications as well, psychotherapy should be sought first. B [1, 9]—The true statements are as follows: A. Antidepressants are useful in diminishing the overall level of anxiety and preventing anxiety or panic attacks, but they have no role in treating acute anxiety. B. Correct—Although often used to treat depression with anxious features, bupropion can have stimulating effects, making it a less attractive choice C. When using SSRI’s, dosages are typically higher when treating anxiety compared to depression D. Duloxetine IS FDA approved for the treatment of generalized anxiety disorder

50

12. D [13]—This individual is suffering from trigeminal neuralgia and first-line treatment is carbamazepine. A. Carbamazepine induces the CYP450 including inducing its own metabolism. As such, it can take greater than 1 month to reach a steady state B. This describes the starting dosage of gabapentin C. This describes topical lidocaine D. Correct. BUN/Cr, CBC w/ diff, LFTs, urinalysis, ophthalmic exam should be performed at baseline, then periodically; serum drug levels should also be monitored. 13. B [14] Prochlorperazine can precipitate neuroleptic malignant syndrome (NMS). NMS is a life-threatening idiosyncratic reaction to antipsychotic drugs characterized by fever, altered mental status, muscle rigidity, and autonomic dysfunction. It has been associated with virtually all neuroleptics, including newer atypical antipsychotics, as well as a variety of other medications that affect central dopaminergic neurotransmission. Although uncommon, NMS remains a critical consideration in the differential diagnosis of patients presenting with fever and mental status changes because it requires prompt recognition to prevent significant morbidity and death. Treatment includes immediately stopping the offending agent and implementing supportive measures, as well as pharmacological interventions in more severe cases. 14. D [15] Topiramate blocks voltage-dependent sodium and calcium channels, inhibits the excitatory glutamate pathway while enhancing the inhibitory effect of GABA, and inhibits carbonic anhydrase activity. The relevant mechanism of action responsible for efficient migraine prophylaxis remains to be determined. 15. C [16] The typical starting dose is 30 mg at dinnertime for a week, then increasing to 60 mg at dinnertime. This individual either did not

7  Psychological Interventions and Pharmacology

increase their dosage or was not told to increase their dosage to 60  mg/day (A). Further, it can take 4–8 weeks at full dosage (B) for clinic effect. Should this individual have side effects, one could consider changing medications to another neuropathic treatment (A). Should they have been at full dosage without effect, there are some individuals who have had benefit at 90 or 120 mg/day of duloxetine, however this is a small subset of individuals. 16. B [17] A. An eighteenth century idea that “a specific sensation, with its own sensory apparatus independent of touch and other senses,” emerged in the nineteenth century, but had been prefigured by the work of Avicenna and Descartes B. Melzak and Wall’s Gate Control Theory stimulated much interest in the multidimensional and subjective aspects of the pain experience. C. Literature on the role of psychologic factors in the experience of pain was summarized in the seminal work of Turk and colleagues that detailed the application of cognitive-behavioral interventions in the management of chronic pain D. The pioneering work of Fordyce et  al. [18] detailed the role that social and environmental factors play in the way an individual expresses pain behaviorally. 1 7. A The boss is exhibiting operant conditioning toward the patient. Activity pacing is a type of operant conditioning as described in A. B describes relaxation. C is most consistent with general care for shoulder arthritis, but is not similar to the boss’s activities. D is a type of coping strategy 1 8. C [19, 20] Hypnosis, whether performed by another individual or via self-hypnosis, requires suggestibility. Theories explaining what occurs during hypnosis fall into two groups. Altered state theories see hypnosis as an altered state of mind or trance, marked by a level of awareness different from the ordinary con-

Answers

scious state. In contrast, nonstate theories see hypnosis as a form of imaginative role enactment. During hypnosis, a person is said to have heightened focus and concentration, however focus and concentration can be seen in other conditions. 19. B [21] While this individual may benefit from multidisciplinary care, the stem most indicates a need for cognitive restructuring. He clearly exhibits a negative attitude and thoughts regarding his pain and describes himself in a worthless manner. Other answer choices could be argued, but cognitive restructuring is the most prudent answer in this scenario. Obviously, while he may have depression, ECT is not the initial treatment for him. 2 0. C [22] A. CBT focuses on challenging and changing unhelpful cognitive distortions (e.g. thoughts, beliefs, and attitudes) and behaviors, improving emotional regulation, and the development of personal coping strategies that target solving current problems. B. Mindfulness is the psychological process of bringing one’s attention to experiences occurring in the present moment, which one can develop through the practice of meditation and through other training C. Correct. Rather than trying to control individual thoughts, feelings, sensations, memories and other private events, ACT teaches them to notice, accept, and embrace their private events, especially previously unwanted ones. D. Cognitive analytic therapy—practitioner aims to work with the patient to identify procedural sequences; chains of events, thoughts, emotions and motivations that explain how a target problem (for example self-harm) is established and maintained. 2 1. D [23] Biofeedback is a technique that teaches an individual to control involuntary physiologic processes by providing detailed feed-

51

back about a process of which typically there is no awareness, e.g. muscle tension (with EMG from site of pain) or skin temperature (with thermistors attached to fingertips). Empirical support for efficacy of biofeedback for pain management is limited to Raynaud’s phenomenon, tension and migraine headaches, and low back pain. An NIH panel reviewed the use of biofeedback in chronic pain and concluded that moderate evidence supported its use, especially in headaches. EMG, electrodermal, skin temperature, and blood-volume-pulse (BVP) biofeedback all demonstrate medium to large effect sizes as treatments for relief of migraine headache. BVP showed the largest treatment effect. Biofeedback is also an effective modality for tension-type headache. 22. C [24, 25] Hypnosis includes an attention-focusing component (similar to relaxation strategies) and a suggestion component delineating specific outcome goals (e.g. pain relief). Hypnosis has been most widely studied and used in treating cancer pain. An NIH panel concluded that strong evidence supports the use of hypnosis in reducing chronic pain due to malignancies. Hypnosis has also been shown to be effective in pain due to irritable bowel syndrome, temporomandibular disorder, and tension headaches. In a review of the effects of hypnosis as a chronic pain treatment, hypnotic interventions consistently produced significant relief in a wide variety of chronic pain disorders. In the 13 trials reviewed, hypnosis was superior to physical therapy and other nonhypnotic interventions. Hypnosis techniques are not well standardized, trials are few and patient numbers are small, so further investigation is required, but hypnotic intervention certainly holds some promise as a pain treatment. 2 3. B [26] Progressive Muscle Relaxation (PMR) is interactive form of relaxation in that the patient is tensing and releasing muscles in a very subtle manner. PMR helps patients dif-

52

ferentiate feelings of tension from relaxation, and learn to apply these skills in painful situations. Stress or pain leads to subtle increases in muscle tension, which can exacerbate pain. Relaxation training attempts to break the pain-muscle tension-­ pain cycle. There is evidence supporting use of relaxation for pain management. PMR has been found to be especially effective for general body pain, specific muscular skeletal pain and tension headaches. 24. A [27, 28] Prochaska and DiClemente’s Transtheoretical Model of Change assesses an individual’s readiness to change and make a healthier behavior. This model views change as a series of stages that an individual goes through in adopting and sticking with a change. The stages are: Pre-contemplation (Not considering change), to Contemplation (Ambivalent about change), Preparation/Determination (Decides to change, but hasn’t started), Action (Involved in change) and Maintenance (Stable change). Ambivalence is a core concept in this model that explains why people don’t change. 2 5. B [26] Operant principles state that providers and family give pain patients positive reinforcement or attempts or success at even small functional goals while ignoring pain behavior. For example, health care providers are neutral or ignore facial grimacing but provides reinforcement to attempts at increasing functioning, like starting a physical therapy program. This can be challenging for physicians as well as family members who need to convey that they understand that the pain condition exists but not perpetuate the sick role.

References 1. Leerman SF, Haythornthwaite J. Psychological evaluation and testing. In: Benzon HT, editor. Essentials of pain medicine. Elsevier; 2018. 2. Fishbain D, Cutler R, Rosomoff H.  Chronic pain associated depression: antecedent or con-

7  Psychological Interventions and Pharmacology sequence of chronic pain? A review. Clin J Pain. 1997;13(2):116–37. 3. Francescangeli J, Karamchandani K, Powell M, Bonavia A.  The serotonin syndrome: from molecular mechanisms to clinical practice. Int J Mol Sci. 2019;20(9):2288. https://doi.org/10.3390/ ijms20092288. Published 2019 May 9. 4. O’Sullivan C, Froyman C.  Nortriptyline safer than amitriptyline? Can Fam Physician. 2018;64(9): 634–6. 5. Dodd S, Malhi GS, Tiller J, et al. A consensus statement for safety monitoring guidelines of treatments for major depressive disorder. Aust N Z J Psychiatry. 2011;45(9):712–25. https://doi.org/10.3109/0004867 4.2011.595686. 6. Cymbalta. 2020. http://www.pdr.net. 7. Gabapentin. 2020. http://www.pdr.net. 8. Milnacipran. 2020. http://www.pdr.net. 9. Buproprion. 2020. http://www.pdr.net. 10. Mirtazipine. 2020. http://www.pdr.net. 11. Toprak SK, Erdogan E, Azap OK.  Mirtazapine-­ induced thrombocytopenia and neutropenia. Letter to the editor. Turk J Hematol. 2012;29:297–8. 12. Heikkila H, Heikkila E, Eisemann M.  Predictive factors for the outcome of a multidisciplinary pain rehabilitation programme on sick leave and life satisfaction in patients with whiplash trauma and other myofascial pain: a follow up study. Clin Rehabil. 1998;12:487–96. 13. Al-Quliti KW.  Update on neuropathic pain treat ment for trigeminal neuralgia. The pharmacological and surgical options. Neurosciences (Riyadh, Saudi Arabia). 2015;20(2):107–14. https://doi. org/10.17712/nsj.2015.2.20140501. 14. Tse L, et  al. Neuroleptic malignant syndrome: a review from a clinically oriented perspective. Curr Neuropharmacol. 2015;13(3):395–406. https://doi. org/10.2174/1570159x13999150424113345. 15. Naegel S, Obermann M.  Topiramate in the prevention and treatment of migraine: efficacy, safety and patient preference. Neuropsychiatr Dis Treat. 2010;6: 17–28. 16. Lunn MPT, Hughes RAC, Wiffen PJ. Duloxetine for treating painful neuropathy, chronic pain or fibromyalgia. 2014. https://www.cochrane.org/CD007115/ NEUROMUSC_duloxetine-­t reating-­p ainful-­ neuropathy-­chronic-­pain-­or-­fibromyalgia. Accessed 2 Sept 2020. 17. Bonica JJ. The management of pain. In: History of pain concepts and therapies, vol. 1. 2nd ed. London: Lea & Febiger; 1990. p. 7. 18. Fordyce, W. E. Behavioral Methods in Chronic Pain and Illness, C. V. Mosby, St. Louis. 1976. 19. Fromm E, Shor RE.  Hypnosis: developments in research and new perspectives. Rutgers; 2009. ISBN 978-0-202-36262-5. Accessed 29 Sept 2020. 20. Kirsch I.  Clinical hypnosis as a nondeceptive placebo. In: Kirsch I, Capafons A, Cardeña-Buelna E, Amigó S, editors. Clinical hypnosis and self-regulation: cognitive-­behavioral perspectives. Washington:

References American Psychological Association; 1999. p. 211–25. 21. Gladding S. Counseling: a comprehensive review. 6th ed. Columbus: Pearson Education Inc.; 2009. 22. Hayes SC, Strosahl KD, Wilson KG.  Acceptance and commitment therapy: the process and practice of mindful change. 2nd ed. New  York: Guilford Press; 2012. p. 240. 23. Andrasik F. What does the evidence show? Efficacy of behavioral treatments for recurrent headaches in adults. Neurol Sci. 2007;28(Suppl 2):S70–7. 24. Pain and Insomnia-NIH Assessment Panel. Integration of behavioral and relaxation approaches into treatment of chronic pain and insomnia. JAMA. 1996;276:313. 25. Elkins G, Jensen MP, Patterson DR. Hypnotherapy for the management of chronic pain. Int J Clin Exp Hypn. 2007;55(3):275–87.

53 26. Haythornthwaite JA, Heinberg LJ.  Psychological intervention in chronic pain. In: Benzon HT, Raja SN, Molloy R, Liu S, Fishman S, editors. Essentials of pain medicine and regional anesthesia. 2nd ed. Philadelphia, PA: Elsevier/Churchill Livingstone; 2005. p. 209–11. 27. Prochaska JO, DiClemente CC.  The transtheoreti cal approach. In: Norcross JC, Goldfried MR, editors. Handbook of psychotherapy integration. 2nd ed. New York: Oxford University Press; 2005. p. 147–71. 28. Prochaska JO, DiClemente CC.  Toward a comprehensive model of change. In: Miller WR, Heather N, editors. Treating addictive behaviors: processes of change. New York: Plenum Press; 1986. p. 3–27.

8

Neurophysiologic Testing

Questions 1. In setting up a new clinic work-flow you wish to have all patients fill out questionnaires prior to their clinic appointment. You choose a questionnaire that is well-validated to evaluate anxiety, depression, marital satisfaction, pain severity, and health locus of control. Which of the following questionnaires did you choose? A. Minnesota Multiphasic Personality Inventory B. Pain Self-Efficacy Questionnaire C. Multidimensional Pain Inventory D. VR-12 2. Which of the following questionnaires is used to evaluate the disability and behavior aspect of pain? A. Brief Pain Inventory B. Pain Locus of Control C. Chronic Pain Acceptance Questionnaire D. Symptom Checklist-90-Revised 3. Which of the following is an 11-point questionnaire used to measure pain-related interference in: general activity, mood, walking ability, normal work, relations with other people, enjoyment of life, and sleep? A. Oswestry Low Back Pain Disability Questionnaire B. Roland-Morris Disability Questionnaire C. Pittsburgh Sleep Quality Index D. Brief Pain Inventory

4. You evaluate an 84-year-old gentleman in clinic who reports his low back pain has greatly interfered with his sleep for several years. He has a somewhat difficult time recalling the onset of his low back pain. He reports both difficulty getting to sleep as well as maintaining sleep. He also reports taking sleeping medications every day for several years, to which his wife retorts that it has only been recently that their primary care provider has provided these medications. Which of the following is the most appropriate way to evaluate his sleep disturbance? A. Pittsburgh Sleep Quality Index administered today and at follow up in 1 month B. Having the patient fill out a sleep diary for the next week and mailing it to the clinic C. Insomnia Severity Index administered today and at follow up in 3 months D. Having his wife fill out the Pittsburgh Sleep Quality Index today and at follow up in 1 month 5. Which of the following is TRUE regarding measurements of psychological function and emotional distress? A. The CES-D has been criticized for under-­ reporting the prevalence of depression in the pain population B. The PHQ-9 shows good sensitivity, specificity, and accuracy in diagnosing depression in chronic pain patients, but cannot be used to track severity of symptoms.

© The Author(s), under exclusive license to Springer Nature Switzerland AG 2021 M. Suer, N. Sehgal, Questions and Answers in Pain Medicine, https://doi.org/10.1007/978-3-030-68204-0_8

55

8  Neurophysiologic Testing

56

C. While most commonly used for anxiety, the GAD-7 has higher sensitivity of detecting PTSD than for generalized anxiety disorder D. Because of sensitivity of detecting depression on the HADS, further questions cannot be omitted if a patient has a score indicating no depression 6. You are designing a clinic protocol and wish to have all patients fill out a questionnaire prior to visits that has the following characteristics: has scales for anger, anxiety, and depression; takes 3–5 min to complete; and captures both positive and negative factors of emotional functioning. Which of the following measures would you select? A. Profile of Mood States (POMS) B. SF-36 C. Symptom Checklist-90-Revised D. Pain Anxiety Symptom Scale 7. Which of these instruments derives its norms from a sample of chronic pain patients? A. (West Haven-Yale) Multidimensional Pain Inventory (MPI) B. Visual Analog Pain Scale (VAS) C. Minnesota Multiphasic Personality Inventory (MMPI) D. McGill Short Form Pain Scale E. SF-36 8. Which psychological measure is the most widely used instrument to assess patients with pain? A. Pain Patient Profile (P3)? B. Symptom Checklist 90, Revised (SCL90R) C. Oswestry Disability Index (ODI) D. Minnesota Multiphasic Personality Inventory-­2 (MMPI-2) E. Multidimensional Pain Inventory (MPI) 9. You are seeing a 61-year-old man for initial consultation for chronic low back pain who endorses drinking “a bit” of alcohol daily. When asked further about the use of alcohol, he immediately retorts: “Doc, don’t try to take away my alcohol! People tell me to cut down, but I just can’t do it. Not right now. Maybe I can when this pain gets better, but not now.” Which of the following is true:

. The patient has alcohol abuse A B. You need to delve further into his alcohol use to determine if he has alcohol abuse C. You should refer him for psychologic evaluation for anger issues D. This is normal behavior

Answers 1. C [1] The MPI is a 56-item survey that evaluates psychosocial, cognitive, and behavior aspects of pain, including pain severity and interference; activity levels; impact of on family relationships and social activities; pain-specific support from spouse or partner; perceived life control; and negative affect 2. A [2] A. Disability and behavior is tested with Brief Pain Inventory, Pain Disability Index, Roland-Morris Disability Questionnaire, Chronic Disability Index, and Oswestry Low Back Disability Questionnaire B. Pain Locus of Control measures locus of control C. Chronic Pain Acceptance Questionnaire measures pain acceptance D. Symptom Checklist-90-Revised measures psychopathology 3. D [2] The Brief Pain Inventory (BPI; 11 items) was developed to measure pain severity and pain-related interference in patients diagnosed with cancer later extended to use in noncancer pain assessment. This scale has been used to demonstrate efficacy of treatment of pain as well. This scale uses an 11-point numeric rating scale (where 0  =  no interference and 10  =  interferes completely) to assess pain-­ related interference in seven areas: general activity, mood, walking ability, normal work including outside the home and housework, relations with other people, enjoyment of life, and sleep. 4. B [3] While A, C, and D each have validated questionnaires to evaluate sleep, they are

Answers

retrospective and the patient in this question likely has a difficult time recalling the true pattern of his sleep. Further, relying on an individual to recall another’s sleep patterns can be misleading. In this situation, it would be best to have the patient fill out a sleep diary to avoid the above factors for obtaining a true knowledge of his sleeping patterns. 5. D [4] A. The CES-D has been criticized for OVER-­ reporting the prevalence of depression in the pain population 8 . B. The PHQ-9 shows good sensitivity, specificity, and accuracy in diagnosing depression in chronic pain patients, but CAN be used to track severity of symptoms. C. While the GAD-7 has been used for diagnosing generalized anxiety disorder, PTSD, social anxiety disorder, and panic disorder; the sensitivity for detecting PTSD is only 66%. The sensitivity for generalized anxiety disorder is 89%. Further, the sensitivities for social anxiety disorder and panic disorder are 74% and 72%, respectively. D. Because of high sensitivity of detecting depression on the HADS, further questions can be omitted if a patient has a score indicating no depression 6 . A [5] The POMS has many advantages over other 9 . scales for measuring emotion associated with pain. The main advantages present include ease of administration, brevity, development of non-psychiatric population, and its design to capture both positive and negative dimensions of emotional function. In particular, it has subscales to measure anxiety, depression, and anger. 7 . A [1, 6–8] The MPI (also known as the WHYMPI) compares patient scores to those of a group of patients with chronic pain, deriving scaled scores on a number of affective and functional domains. The other instruments do not use chronic pain patients for their norms. The VAS is a numerical rating of pain intensity from 0 (no pain) to 10 (worst pain

57

imaginable) and is not compared to any normative sample. The MMPI is a lengthy multiple-­choice questionnaire examining different dimensions of personality and used in the diagnosis of mental illness. Its norms are derived from the general population. The McGill Short Form is designed to quantify different aspects of the pain experience; it does not use chronic pain patients for norms. The SF-36 measures function and health-­related quality of life, not pain. D [8, 9] The Minnesota Multiphasic Personality Inventory-2 (MMPI-2) is the most broadly used and validated scale. It is a self-report instrument that asks patients to answer 567 questions as true or false and attempts to classify patients according to personality types. Analysis of response patterns yields a psychological profile. As this tool was first used in psychiatric patients who do not have chronic pain, many of the questions answered by chronic pain patients would lead to their receiving a high score on the hypchondriacal scale. An appropriate use of this scale is to help formulate hypotheses with regard to the presence of comorbid psychopathology and personality characteristics that have the potential to influence the patient’s adaptation to the pain experience and possibly interfere with treatment. A [10] The CAGE questionnaire has been shown to successfully elicit alcohol and drug abuse history. Two positive responses to the CAGE questions indicate substance abuse. In his response, he has answered positive to trying to Cut down as well as feeling Annoyed. Asking further about a Guilt with drinking or needing an Eyeopener would not provide more diagnostic information about substance abuse. It could, however, provide some insight into the depth of his use. Further interrogating about an issue, he clearly has issues with could also damage the physician-patient relationship. You could use this opportunity to build r­ apport and then address the alcohol further once you have established a good working relationship with the patient.

58

References 1. Kerns RD, Turk DC, Rudy TE. The West Haven-Yale Multidimensional Pain Inventory (WHYMPI). Pain. 1985;23:345–56. 2. Cleeland CS, Ryan KM. Pain assessment: global use of the Brief Pain Inventory. Ann Acad Med Singap. 1994;23:129–38. 3. Leerman SF, Haythornthwaite J. Psychological evaluation and testing. In: Benzon HT, editor. Essentials of pain medicine. Elsevier; 2018. 4. Zigmond AS, Snaith RP.  The hospital anxiety and depression scale. Acta Psychiatr Scand. 1983;67:361–70. 5. McNair D, Lorr M, Doppleman L. POMS manual for the profile of mood states. San Diego, CA: Educational and Industrial Testing Service; 1971.

8  Neurophysiologic Testing 6. Melzack R.  The short-form McGill Pain Questionnaire. Pain. 1987;30(2):191–7. 7. Ware JE Jr. SF-36 health survey update. Spine. 2000;25(24):3130–9. 8. Vendrig AA. The Minnesota Multiphasic Personality Inventory and chronic pain: a conceptual analysis of a long-standing but complicated relationship. Clin Psychol Rev. 2000;20(5):533–59. 9. Riley JL III, Robinson ME.  Validity of MMPI-2 profiles in chronic back pain patients: differences in path models of coping and somatization. Clin J Pain. 1998;14(4):324–35. 10. Mayfield D, McLeod G, Hall P. The CAGE questionnaire: validation of a new alcoholism screening instrument. Am J Psychiatry. 1974;131:1121–3.

Part III Common Pain Syndromes

9

Fibromyalgia and Myofascial Pain

Questions 1. Which of the following is true regarding myofascial trigger points? A. Trigger points always cause pain at rest B. EMG of the myofascial trigger points reveals normal insertional activity and a myotonic pattern on muscle activation C. Can lead to peripheral sensitization but not central sensitization D. Acetylcholine has been implicated in the motor phenomena of myofascial trigger points 2. A 34-year-old college professor is seen in the pain clinic for evaluation of chronic pain. She has multiple points of pain on palpation of the muscles in her upper back and neck. Which of the following would most reliably point to a diagnosis of myofascial pain rather than fibromyalgia? A. Multiple muscles with painful areas on palpation B. Taut muscle band without twitch upon palpation C. Pain referred toward the head with palpation of the muscles of the neck D. Area of reddened skin following palpation of the tender area E. All of the above 3. You evaluate a 31-year-old triathlete in clinic with chronic pain in his right upper trapezius. He notes the pain is always worst when he

spends more time on his bicycle. On exam, you note a taut band in his bilateral trapezius, both of which cause radiating of the pain into his head. His right levator scapula feels contracted. What is the next step in management of his pain? A. Refer him to physical therapy to include myofascial treatments and dry needling B. Perform trigger point injections today in clinic C. Order botulinum toxin injections into his upper trapezii and right levator scapulae D. Start him on fluoxetine 4. You decide to perform trigger point injections on an otherwise healthy 32-year-old female. Which of the following NEED NOT be included in the consent for injection of the upper back musculature? A. Localized pain during injection B. Nerve block C. Pneumothorax D. Infection E. All of the above should be included 5. You are performing ultrasound-guided injections into the levator scapulae. What is true following use of ultrasound guidance for trigger point injections? A. Trigger point shows areas area of hyperechoic tissue B. Cannot be reliably used to differentiate trigger point from normal muscular texture

© The Author(s), under exclusive license to Springer Nature Switzerland AG 2021 M. Suer, N. Sehgal, Questions and Answers in Pain Medicine, https://doi.org/10.1007/978-3-030-68204-0_9

61

9  Fibromyalgia and Myofascial Pain

62

C. Allows visualization of the muscle twitch D. Has little to no advantage for treatment over palpation-based injections 6. Which is the most correct statement about botulinum toxin for trigger point injections in myofascial pain? A. Is more effective than “dry needling” B. Is as effective as lidocaine injection C. Prolongs lidocaine effect when used in combination D. Is a second line treatment for refractory tender points E. Should not be used because it causes myonecrosis 7. A 53-year-old man with chronic right hip pain (pointing toward the front of the hip) is evaluated in clinic. With his right leg hanging off the examination table and left leg flexed, he has concordant pain in the right hip. Hip internal and external range of motion and circumduction are normal for age without pain. What is the next step in treatment? A. Physical therapy including dry needling B. Intra-articular hip joint injection under fluoroscopic or ultrasound guidance C. Lumbar spine MRI to evaluate for radiculopathy D. Botulinum toxin injections into the iliopsoas 8. You wish to perform an ultrasound-guided anterior scale injection. Where should the needle be placed in the following picture (Fig. 9.1)? A. 1 B. 2 C. 3 D. 4 9. Which of the following substances has been shown to be decreased in patients with fibromyalgia A. Substance P B. 5-HT C. IL-6 D. ACTH 10. A 43-year-old male is diagnosed with fibromyalgia. In review of the chart, you find the following labs: ALT 90  units/mL, AST

Fig. 9.1

78  units/mL, GFR 110. Which medication should be chosen for a first medication trial? A. Pregabalin B. Nortriptyline C. Duloxetine D. Tizanidine 11. Which of the following treatments exhibits the least amount of evidence for the treatment of fibromyalgia? A. CBT B. Tizanidine C. Aerobic exercise D. Calcium channel blockers 12. Which of the following individuals meets diagnostic criteria for diagnosis of fibromyalgia? A. Widespread Pain Index score of 6 and Symptom Severity score 7 with symptoms present for 1 year B. Widespread Pain Index score of 6 and Symptom Severity score 12 with symptoms present for 1 month C. Widespread Pain Index score of 9 and Symptom Severity score 4 with symptoms present for 17 years D. Widespread Pain Index score of 4 and Symptom Severity score 10 with symptoms present for 1 year 13. A 43-year-old female is evaluated for fibromyalgia. In review of her chart, EKG shows normal sinus rhythm with QT interval of 505  ms. She has normal renal and hepatic function. She regularly takes loperamide,

Answers

propranolol, and simvastatin. Which of the following medications would be your first step in management? A. Cyclobenzaprine B. Nortriptyline C. Pregabalin D. Duloxetine 14. Individuals with prolonged QT are at risk of which cardiac abnormality? A. Ventricular tachycardia B. Atrial fibrillation C. Atrial flutter D. Conduction block 15. A 28-year-old female is evaluated for treatment options for fibromyalgia. EKG shows no abnormalities with QT interval of 402 ms. She has no hepatic or renal abnormalities. She was recently seen by her ophthalmologist and diagnosed with early glaucoma. Which medication is a reasonable first choice treatment for her fibromyalgia A. Fluoxetine B. Milnacipran C. Amitriptyline D. Gabapentin 16. Which of the following factors is most correlated with the analgesic efficacy of anti-­ depressant medications? A. Serotonin reuptake inhibition B. Norepinephrine reuptake inhibition C. Dopaminergic activity D. Side effect profile 17. Which of the following muscle relaxants works at the muscular level? A. Baclofen B. Tizanidine C. Dantrolene D. Carisoprodol 18. Which of the following is contained in EMLA cream A. Marcaine B. Prilocaine C. Tetracaine D. Iontocaine 19. Which of the following muscle relaxants has the highest potential for addiction A. Cyclobenzaprine B. Baclofen

63

C. Carisoprodol D. Tizanidine 20. Which of the following is the most common cause of thoracic outlet syndrome? A. Venous B. Arterial C. Neurogenic D. Nonspecific 21. A 56-year-old female is being evaluated for temporomandibular joint dysfunction. You evaluate her maximal mouth opening—measured as the distance between the juxtaposed edges of the upper and lower incisors. A distance below which mark would categorize her as reduced mouth opening? A. 20 mm B. 30 mm C. 40 mm D. 50 mm 22. Diffuse musculoskeletal and bone pain have been associated with a deficiency of which important nutritional molecule? A. Vitamin B6 B. Vitamin D C. Selenium D. Vitamin C E. Iron

Answers 1. D [1] A. Active trigger points cause pain at rest whereas latent trigger points cause pain only with palpation or other stimulation B. Abnormal spontaneous electrical activity is present at the site of TrPs, with excessive ACh release creating endplate noise seen on electrophysiological studies at the neuromuscular junction. C. In a sensitized state, nociceptors spontaneously discharge with a lower threshold to painful stimulation and also exhibit discharge to nonpainful stimuli. Over time, this heightened abnormal peripheral sensory input creates a state of central neuronal sensitization. D. Correct

64

2. C [2] A. While in the past, fibromyalgia reports 11 out of 18 areas of pain upon palpation, this is no longer a diagnostic criterion. Further, it is not uncommon to have multiple areas of pain with only myofascial pain. While this may be subjectively true, it is not the best answer available. B. The localized twitch response upon palpation, while very specific if present, is not a sensitive test for myofascial pain vs fibromyalgia. It can be quite difficult to elicit with only palpation and can more reliably be noted with trigger point injections C. The most specific finding of trigger points over fibromyalgia is the presence of referred pain D. This can be seen with any area of restriction whether muscular, fascial, tendon injury, etc. It is not specific for trigger points and cannot be reliably used to distinguish fibromyalgia from myofascial pain. 3. A [1] A. Myofascial pain is best treated in a multidisciplinary fashion including both physical therapy and dry needling (or trigger point injections) B. While performing trigger point injections in clinic is not incorrect, one should note the chronic nature of the issue and address it in a multidisciplinary fashion. He notes his pain is worse on the bicycle leading one to believe it is a biomechanical factor causing his pain. This needs to be addressed and he may benefit from specific stretching and strengthening exercises to address this possible imbalances. C. Botulinum toxin injections have not shown to be superior to dry needling or trigger point injections with local anesthetic for treatment of myofascial trigger points. D. He does not have fibromyalgia and fluoxetine does not have a role in the initial treatment of his symptoms.

9  Fibromyalgia and Myofascial Pain

4. E [3] All of the above need to be included in the consent for trigger point injections into this area. Possible complications include bleeding, hematoma, nerve block or other neurologic symptoms (depending on area injected), infection. There are case reports of pneumothorax when performing trigger point injections into the cervicothoracic region using a needling technique. For injections in this area as well as thoracic in particular, a pincer grip to bring the affected tissue away from the rib cage should be utilized. 5. C [4, 5] A. Trigger point shows areas area of HYPOechoic tissue B. Can be reliably used to differentiate trigger point from normal muscular texture as trigger point shows as hypoechoic tissue within an otherwise normal muscle architecture C. Correct D. There are multiple advantages of use of ultrasound for trigger point injections including avoidance of neurovascular structures, visualization of the myofascial trigger point, visualization of the twitch response, accurate needle placement in deep muscle, and confirmation of local anesthetic infiltration between fascial planes. It can also be used to perform nerve blocks (e.g., spinal accessory nerve for diagnosis of trapezius muscle-related myofascial pain). 6. B [6, 7] Botulinum toxin (BTX), a chemodenervation agent producing sustained muscle relaxation, has been employed as a treatment for injection into painful myofascial trigger points (TrP). Available evidence suggests that most studies of BTX in myofascial pain have involved few patients and been poorly controlled. BTX injection into TrP is effective at relieving pain, but no more so than lidocaine or dry needling. It is not combined with any agent. Bupivacaine injection is associated with myonecrosis.

Answers

A small 2004 study compared BTX Type A (n = 9), lidocaine (n = 10), and dry needling (n = 10) for myofascial TrP in a single-­blind prospective study with followup 4  weeks post-injection. This study demonstrated some superiority of BTX-A and lidocaine over dry needling for improving quality of life and reducing pain. The authors concluded that lidocaine is the preferred treatment for myofascial TrP because of the expense of BTX and its similar effectiveness to lidocaine. Later studies have shown similar results. 7. A [8] This individual is likely suffering from iliopsoas syndrome. The physical examination maneuver used, the Gaenslen’s test, is often used for SI joint dysfunction (patient experiences posterior/SI pain) can be used to iliopsoas dysfunction if they experience anterior hip pain. A. Physical therapy including dry needling is the best first-step management for iliopsoas syndrome B. There is no evidence that he has hip osteoarthritis or intra-articular pathology. Further, should this be suspected, the next step in management would be to order basic imaging of the hip to confirm the diagnosis. Had obtaining imaging been an answer choice, it likely would be the direction most of us would go clinically. However, one would not simply go straight to an injection in this individual. C. He does not report back pain and does not show evidence of radicular pain based on the question stem. D. Botulinum toxin injections into the iliopsoas can be done after failure of conservative treatment trial. There exists only one study showing the superiority of botulinum toxin injections over steroid + local anesthetic. 8. A A. Anterior scalene B. Sternocleidomastoid C. C6 nerve root D. Middle scalene

65

9. B [9] Also note that glutamate is often found to be elevated in patients with fibromyalgia and GABA decreased. A. Substance P is elevated B. 5-HT is decreased C. IL-6 is elevated D. ACTH is elevated; however, the level of cortisol has not been shown to rise commensurately 10. A [10] A. No medication dosage adjustments are required for individuals with hepatic impairment for pregabalin B. Since nortriptyline is substantially metabolized, initiate dosage cautiously in patients with hepatic impairment. C. Avoid use in patients with any chronic hepatic disease or cirrhosis, due to an increased risk for hepatotoxicity. D. Not recommended for patients with moderate to severe hepatic impairment. In patients with mild hepatic impairment, begin with 5  mg PO dose and titrate upward slowly; these patients may require less frequent dosing than the three times daily schedule recommended for adults 11. B [10] A. CBT has been shown to be beneficial in one-on-one, group, and internet settings. Level of evidence is 1,A B. Tizanidine has only been shown to be of benefit in 1 small randomized control trial. It is the only option listed that does not have 1, A level of evidence. C. Aerobic exercise, specifically graded exercise has Level 1, A evidence. Strengthening and stretching has also been show to be of value. D. Gabapentinoids such as pregabalin have Level 1, A evidence for treatment of fibromyalgia. 12. D [11] Criteria are as follows A. WPI > 7 and SS > 5 OR WPI of 3–6 and SS > 9

66

B. Symptoms present at similar level for at least 3 months C. No disorder present that would otherwise explain the pain 13. C [12] This individual has prolonged QT and all medication and medication combinations that can further prolong the QT interval should be avoided. A. Cyclobenzaprine can prolong the QT interval B. Nortriptyline can prolong the QT interval C. Correct D. Duloxetine in combination with loper amide can prolong the QT interval further and use should be avoided. The plasma concentration of loperamide, a CYP2D6 substrate, may be increased when administered concurrently with duloxetine, a CYP2D6 inhibitor. 14. A [13] Specifically, they are at risk for torsades de pointes which is a polymorphic ventricular tachycardia. Although certainly not an exhaustive list, common pain medications that can cause QT prolongation include amitriptyline, nortriptyline, methadone, tizanidine, and tramadol. Others include omeprazole and many SSRI’s and SNRI’s in particular. 15. D [14] Each of the following answers, except gabapentin has been shown to exacerbate glaucoma. Pregabalin has a mild incidence of increased intraocular pressure, however this has not been reported with gabapentin. Despite not being FDA-approved for fibromyalgia, it is the only option listed that does not have reports of increased intraocular pressure. 16. B [15] While each of these are factors included in the ability of a medication to effect pain, the more effective medications tend to have a higher norepinephrine to serotonin reuptake inhibition ratio. However, pure norepinephrine reuptake inhibitors have not shown to be effective analgesics, thus a combination of serotonin and norepinephrine seem to have

9  Fibromyalgia and Myofascial Pain

a synergistic effect. Side effect profile is often utilized clinically to determine which medication to try first but is not a primary determinant in the in the medication’s innate ability to effect pain. 17. C [16–19] A. Baclofen is believed that the drug works mainly at the level of the spinal cord to block polysynaptic afferent pathways and, to a lesser extent, monosynaptic afferent pathways although the mechanism is not totally clear. B. Tizanidine, which is structurally and pharmacologically related to clonidine, is a central-acting alpha2-adrenergic agonist which acts at presynaptic receptors. C. Dantrolene directly interferes with calcium ion release from the sarcoplasmic reticulum within skeletal muscle cells. D. The actions of carisoprodol are related to a CNS mechanism and not to a direct effect on skeletal muscle. Carisoprodol appears to interrupt neuronal communication within the reticular formation and spinal cord although the mechanism isn’t completely certain 1 8. B [20] EMLA cream is made of 2.5% lidocaine and 2.5% prilocaine 1 9. C [21] The therapeutic effect of Soma is effectively to interfere with pain sensation signaling that occur between peripheral pain receptors (nerves) and certain areas of the central nervous system (the brain). In addition to the modification of pain signaling, Soma and its main metabolite (meprobamate) exert mild sedative effects. Many Soma users find the sedative effect of carisoprodol to be pleasant which, in turn, can drive a compulsion for continued use. When taken as prescribed, it is generally viewed as safe. For that reason, it has not yet made it onto the DEA’s controlled substances list. However, it can be addictive. In fact, some states have listed Soma as a scheduled substance.

References

20. C [22, 23] A. Neurogenic TOS includes disorders produced by compression of components of the brachial plexus nerves. The neurogenic form of TOS accounts for 95% of all cases of TOS. B. Arterial TOS is due to compression of the subclavian artery. This is less than one percent of cases C. Venous TOS is due to compression of the subclavian vein. This makes up about 4% of cases. 21. C [24] Normal mouth opening distance is often cited as 40  mm (or more commonly, the ability to insert three fingers—index, middle, ring—aligned vertically into the open mouth) between the upper and lower incisors. A distance less than 40  mm would qualify as reduced mouth opening. 22. B [25] Musculoskeletal and bone pain are known complications of hypovitaminosis D.  A study measuring vitamin D levels in patients with nonspecific, diffuse musculoskeletal pain found vitamin D deficiency in 93% and severe deficiency (≤8  ng/mL) in 28%. Patients with diffuse musculoskeletal pain should be screened for vitamin D deficiency. Caveats: The true cutoff levels for hypovitaminosis D are still not completely defined, and hypovitaminosis D has not been definitively linked to any specific pain condition.

References 1. Bordoni B, Sugumar K, Varacallo M. Myofascial pain. [Updated 13 Sept 2020]. In: StatPearls [Internet]. Treasure Island, FL: StatPearls Publishing; 2020. https://www.ncbi.nlm.nih.gov/books/NBK535344/. 2. McMahon SB, Koltzenburg M, editors: Wall and Melzack’s textbook of pain. 5th ed. Philadelphia: Elsevier; 2006. p. 669–81. 3. Hammi C, Schroeder JD, Yeung B.  Trigger point injection. [Updated 31 Jul 2020]. In: StatPearls [Internet]. Treasure Island, FL: StatPearls Publishing; 2020. https://www.ncbi.nlm.nih.gov/books/ NBK542196/.

67 4. Ricci V, Özçakar L.  Ultrasound imaging of the upper trapezius muscle for safer myofascial trigger point injections: a case report. Phys Sportsmed. 2019;47(3):247–8. 5. Finlayson RJ.  Ultrasound guidance for trigger point injections: gold standard or fool’s gold? Reg Anesth Pain Med. 2017;42(3):279–80. 6. Rudin NJ.  Evaluation of treatments for myofascial pain syndrome and fibromyalgia. Curr Pain Headache Rev. 2003;7:433–42. 7. Kamanli A, et al. Comparison of lidocaine injection, botulinum toxin injection, and dry needling to trigger points in myofascial pain syndrome. Rheumatol Int. 2005;25(8):604–11. 8. Dydyk AM, Sapra A.  Psoas syndrome. [Updated 29 Jun 2020]. In: StatPearls [Internet]. Treasure Island, FL: StatPearls Publishing; 2020. https://www.ncbi. nlm.nih.gov/books/NBK551701/. 9. Dadabhoy D, Crofford LJ, Spaeth M, Russell IJ, Clauw DJ.  Biology and therapy of fibromyalgia. Evidence-based biomarkers for fibromyalgia syndrome. Arthritis Res Ther. 2008;10(4):211. https:// doi.org/10.1186/ar2443. 10. Clauw DJ.  Fibromyalgia: a clinical review. JAMA. 2014;311(15):1547–55. https://doi.org/10.1001/ jama.2014.3266. PMID: 24737367. 11. American Psychiatric Association. Diagnostic and statistical manual of mental disorders: DSM-5. Amer Psychiatric Pub Inc.; 2013. 991 p. 12. Schächtele S, Tümena T, Gaßmann KG, Fromm MF, Maas R.  Co-prescription of QT-interval prolonging drugs: an analysis in a large cohort of geriatric patients. PLoS One. 2016;11(5):e0155649. https:// doi.org/10.1371/journal.pone.0155649. Published 2016 May 18. 13. Beach SR, Celano CM, Noseworthy PA, Januzzi JL, Huffman JC.  QTc prolongation, torsades de pointes, and psychotropic medications. Psychosomatics. 2013;54(1):1–13. https://doi.org/10.1016/j. psym.2012.11.001. PMID: 23295003. 14. Gabapentin. 2020. https://www.pdr.net/drug-­summary/ Neurontin-­gabapentin-­2477.4218. 15. Jasmin L, Tien D, Janni G, Ohara PT. Is noradrenaline a significant factor in the analgesic effect of antidepressants? Pain. 2003;106(1):3–8. https://doi. org/10.1016/j.pain.2003.08.010. 16. Dantrolene. 2020. https://www.pdr.net/drug-­ summary/Dantrium-­C apsules-­d antrolene-­s odium­1213#15. 17. Baclofen. 2020. https://www.pdr.net/drug-­summary/ Baclofen-­baclofen-­1058.3913#15. 18. Tizanidine. 2020. https://www.pdr.net/drug-­summary/ Tizanidine-­tizanidine-­3138#14. 19. Carisoprodol. 2020. https://www.pdr.net/drug-­ summary/Soma-­carisoprodol-­2128.4395#14. 20. Lidocaine/prilocaine. 2020. https://www.pdr.net/ drug-­summary/Oraqix-­lidocaine-­prilocaine-­2576. 21. American Addiction Centers Editorial Staff. The effects of soma use. In: Lautieri A, editor. https://drugabuse.com/soma/effects-­use/.

68 22. Fugate MW, Rotellini-Coltvet L, Freischlag JA. Current management of thoracic outlet syndrome. Curr Treat Options Cardiovasc Med. 2009;11(2):176–83. 23. Kuhn JE, Lebus GF, Bible JE.  Thoracic outlet syndrome. J Am Acad Orthop Surg. 2015;23(4):222–32. 24. Zawawi KH, Al-Badawi EA, Lobo SL, Melis M, Mehta NR.  An index for the measurement of nor-

9  Fibromyalgia and Myofascial Pain mal maximum mouth opening. J Can Dent Assoc. 2003;69(11):737–41. 25. Plotnikoff GA, Quigley JM.  Prevalence of severe hypovitaminosis D in patients with persistent, nonspecific musculoskeletal pain. Mayo Clin Proc. 2003;78(12):1463–70.

10

Central Pain

Questions 1. Which of the following pathways is a key component to the pathophysiology of central post-stroke pain syndrome? A. Corticobulbar pathway B. Dorsal column pathway C. Corticospinal pathway D. Spinothalamocortical pathway 2. A 62-year-old female nursing home patient with history of diabetes presents for evaluation of 3-day history of left leg and foot pain described as aching, deep, and squeezing. She suffered a right-sided thalamic stroke 1  month prior resulting in numbness and sensory loss on the left side. What is the next step in the management of this patient? A. Lower extremity ultrasound B. Lumbar MRI C. Hemoglobin A1c and prescribe gabapentin D. No further tests needed. Prescribe nortriptyline 3. Stimulation of which of the following areas has NOT shown to provide reduction in severity of chronic post-stroke pain? A. Sensory cortex B. Periventricular grey C. Thalamus D. Motor cortex 4. Which of the following statements regarding central pain is FALSE?

A. The leading cause of central pain originating from the brain is stroke B. Classically described central pain disorders are more commonly seen in women than in men C. The elderly are more affected in the cases of poststroke pain, while SCI and MS pain tend to affect younger patients D. The highest prevalence of central pain is reported in cases of lesions in the spinal cord, medulla, and ventroposterior part of the thalamus 5. Which of the following statements regarding spinal cord stimulator treatment for central pain is TRUE? A. Dorsal columns should be functional above the level of injury to produce paresthesia B. Patients must experience total pain relief during trial stimulation are potential candidates for an implant C. If treatment fails with spinal cord stimulation, further treatment with deep brain stimulation should not be pursued D. Spinal cord stimulation has shown use with central pain of cranial origin 6. After evaluating a 38-year-old woman with central pain secondary to multiple sclerosis, you decide to proceed with intrathecal ziconotide treatment. Which of the following is a contraindication for intrathecal ziconotide?

© The Author(s), under exclusive license to Springer Nature Switzerland AG 2021 M. Suer, N. Sehgal, Questions and Answers in Pain Medicine, https://doi.org/10.1007/978-3-030-68204-0_10

69

70

10  Central Pain

. Use of oral opiates A screws. Nine  months later he presents B. Depression controlled with duloxetine with persistent pain in the left lateral leg C. Cervical stenosis and dorsolateral aspects of his left foot. D. Renal impairment Pain is constant, burning, tingling and 7. Which of the following is least likely to there are spontaneous pain exacerbations. occur with a left hemisection of the spinal He reports increase in pain with touch, cord from a fracture of the T1 vertebra? on contact with clothing, and accidental A. Loss of position and vibration on the left bumps against the left lateral leg cause B. Loss of pinprick and temperature on the excruciating electric shock like pain. right 10. Based on this description and distribution C. Loss of two point discrimination on the of this patient’s pain, the nerve that is most left likely to be involved is? D. Claw hand deformity of left hand A. Sural E. A positive Babinski sign on the left B. Superficial peroneal 8. Which of the following statement is true C. Tibial regarding pain pathways at the thalamic and D. Sciatic cortical levels? 11. Pain described as burning is associated with A. Discriminative aspects of pain are related spontaneous activity in the nerve fibers clasto areas 3, 1 and 2 (primary somatosensified as? sory cortex) A. A-beta B. Affective-motivational aspects of pain B. A-delta are related to cingulate cortex, insula and C. C-fibers prefrontal cortex D. C-fibers C. Lesion of the primary sensory cortex do 12. Neurosensory examination reveals decreased not extinguish emotional aspects of pain sensory perception to vibration, worsening D. Affective, emotional aspects of pain are of pain with light brush and tingling on light conveyed to the cortex via intralaminar tap behind fibular head. Tingling sensation and mediodorsal thalamic nuclei on tapping at the fibular head in this patient E. All of the above suggests evoked activity in which nerve 9. The phenomenon of central sensitization is fibers? primarily mediated by A. A-alpha A. NMDA-dependent “windup” at Pacinian B. A-beta corpuscles C. A-delta B. Increased circulating norepinephrine D. C-fibers C. Sensitization of dorsal horn wide-­ 13. Worsening of pain due to light brush would dynamic-­range neurons signify presence of which of the following D. Sensitization of GABAergic interneu processes: rons A. Transduction E. Peripheral release of substance P at noci B. Peripheral transmission ceptive terminals C. Central sensitization _________________________________ D. Descending inhibition Use the following question stem to 14. This patient participated in a research study answer questions 10 through 14. that studied the effect of experimental pain A 24-year-old man was involved in on his leg pain. A heat probe at 45  °C was a skateboarding accident, when he susapplied on the affected left limb and pain tained complex fracture of left tibia intensity ratings were recorded before and requiring stabilization with plates and after immersion of the unaffected right limb

Answers

in a hot water bath at 50 °C. Pain intensity ratings from the heat probe on the left limb did not decrease after hot water bath immersion of right limb. This suggests: A. Facilitated thermal hyperalgesia B. Block of peripheral transmission C. Synaptic transmission at dorsal horn D. Failure of descending inhibition __________________________________ 15. The most common cause of persistent spinal cord pain is: A. Trauma B. Post-surgery related phenomena C. Neoplasm D. Vascular disease E. Congenital 16. The most likely cause of delayed-onset, vague, neuropathic pain of CNS origin in a patient with spinal cord injury is: A. Partial damage to spinal cord B. Damaged visceral afferents C. Cauda equina injury D. Spinal cord syrinx E. Segmental deafferentation 17. Which of the following is TRUE of pain states in multiple sclerosis (MS)? A. Acute pain syndromes occur in approximately 50% patients with MS B. Trigeminal neuralgia may be the first manifestation of MS in older patients C. Continuous burning lower limb pain occurs commonly in MS D. Opioid therapy is contraindicated because of risk of psychologic dependence E. Steroids provide first line treatment for paroxysmal pain in MS. 18. Which of the following is true of central post-stroke pain (CPSP)? A. Refers to any pain symptoms in patients with history of stroke B. May be peripherally or centrally mediated C. Manifests as pain and sensory abnormalities in affected body part D. Results from spasticity and paresis in the shoulder E. Has a psychogenic basis

71

19. Regarding central post stroke pain (CPSP): 1. The majority of patients with CPSP have a lesion in the thalamus 2. Damage to the spinothalamocortical pathway plays a significant role in CPSP 3. Onset of pain is typically delayed for over a year after the stroke 4. Altered temperature sensibility is a common feature in CPSP patients A. 1 and 3 B. 2 and 4 C. 1, 2 and 3 D. All of the above 20. A correct statement regarding treatment options for central post stroke pain (CPSP) is: A. Amitriptyline is the first line of treatment B. Spinal cord stimulation is an effective alternative C. Opioids are contraindicated D. Lamotrigine has been found to be ineffective E. Motor cortex stimulation has no demonstrated utility

Answers 1. D [1] For diagnosis of CPSP, a lesion in the spinothalamocortical pathway is necessary and can develop after cortical, subcortical, thalamic, and lateral brainstem strokes. There exist multiple theories to the pathophysiology of CPSP, but all depend upon this concept. 2. A [2] Diagnosis of chronic post-stroke pain is a diagnosis of exclusion as multiple pain conditions are common in the acute and subacute stages after stroke. The patient in this vignette has risk factors for DVT. Given the acuity of onset, a lower extremity ultrasound is warranted. Other diagnostic criteria include pain within a distinct neuroanatomically plausible distribution, history suggestive of stroke, indication of a distinct neuroanatomic distribution by clinical examination, and indication of a relevant vascular lesion on imaging.

72

3. A [3, 4] In patients with treatment failure, deep brain stimulation (DBS) of the tactile relay nucleus of the thalamus or the lemniscal radiations offers potential treatment. There is some suggestion that paresthesia-producing DBS alleviates steady neuropathic pain. Periventricular/periaqueductal gray (PVG/PAG) DBS is appropriate for nociceptive pain. Stimulating the motor cortex offers a new target for the neuromodulation of central pain as demonstrated Yamamoto and colleagues. They concluded that patients whose pain was diminished by thiamylal and ketamine, but not by morphine, respond best to MCS. 4. B [2, 5, 6] The elderly are more affected in the cases of post-stroke pain, while SCI and MS pain tend to affect younger patients. 5. A [3, 6] A. Dorsal columns should be functional above the level of injury to produce paresthesia. B. Patients must experience at least 50% pain relief during trial stimulation are potential candidates for an implant. One should also evaluate for functional improvement. C. In patients with treatment failure, deep brain stimulation (DBS) of the tactile relay nucleus of the thalamus or the lemniscal radiations offers hope D. Even with positive trial, spinal cord stimulation has not shown use with brain-origin central pain 6. C [7] A. Use of opioid medications can lead to increase incidence of dizziness and confusion but it does not interact with opioid receptors and does not potentiate opioid side effects including respiratory depression. B. Patients with a history of psychiatric illness should be screened however depression controlled with either pharmacologic or non-pharmacologic means is not a contraindication. Individuals with uncontrolled depression, bipolar disorder, suicidal ideation, and schizophrenia are poor candidates for intrathecal therapies.

10  Central Pain

C. Of the options listed, only cervical stenosis is a contraindications adequate circulation of the cerebrospinal fluid is needed to allow for medication distribution. D. While there are not specific guidelines for dosage adjustments with renal impairment, it does not appear that adjustments are needed. 7. D [8] Hemisection of the spinal cord in the sagittal plane results in Brown-Sequard syndrome, which is characterized by: • Ipsilateral loss of position, vibration, and two-­point discriminatory sensation (due to dorsal column injury) below the cord level of the lesion • Contralateral loss of pain and temperature sensation (due to spinothalamic tract injury) starting one or two dermatomes below the cord level of the lesion • Ipsilateral weakness and upper motor neuron signs (due to pyramidal tract injury) below the cord level of the lesion Cord levels and vertebral levels are spatially different. In the cervical cord, the cord segment is one level cranial to the samenumbered vertebra; in the thoracic cord, the cord segment is 2–3 levels cranial to the same-numbered vertebra. In the case of a T1 fracture, therefore, the spinal cord injury level is T3 or T4, not T1. Intrinsic hand muscles (C8–T1) will be spared, so no “claw hand” will result. 8. E [8] Discriminative aspects of pain sensation are related to the primary somatosensory cortex (areas 3, 1, and 2), which receives signals via the ventroposteromedial (VPM) and ventroposterolateral (VPL) thalamic nuclei. Affective/emotional aspects of pain sensation are related to activity in the cingulate cortex, prefrontal cortex and insula, which receive input from the intralaminar and mediodorsal thalamic nuclei. A lesion in primary sensory cortex therefore will not affect emotional aspects of the pain experi-

Answers

ence; conversely, a cingulate lesion will not affect pain discrimination and localization. 9. C [9] NMDA-dependent “windup” is a phenomenon seen in the dorsal horn, not the Pacinian corpuscles, which are located at sensory nerve endings. Increased circulating norepinephrine is a possible contributing mechanism to sympathetically-maintained pain syndromes. GABAergic interneurons would have an inhibitory, not excitatory effect. Peripheral release of substance P at nociceptive terminals is associated with neurogenic inflammation. 10. B [10–14] The nerve distribution matches the superficial peroneal nerve. In addition to providing sensation along the anterolateral aspect of the leg and the dorsum of the foot (except the first web space), it also provides innervation to the peroneus longus and brevis muscles. 11. D [10–14] The burning sensation is provided by C-fibers, which are small in diameter and unmyelinated and have lower conduction velocity. C-fibers respond to various stimuli including thermal, mechanical, and chemical. 12. B [10, 12–15] This sensation is being transmitted by A-beta type nerve fibers. While the majority of these are dedicated to touch, a fraction can also transmit pain. 13. C [10, 12–15] This phenomenon, called allodynia, is a process of central sensitization. A common example of how to think of this when discussing patients is even light touch of a sunburn causing pain (although different mechanism, this is the best way to explain light touch causing pain many times in lay terms.) 14. D [10, 12–15] Multiple regions in the brain—midline periaqueducal gray-rostral ventromedial medulla (PAG-RVM) system, and the more lateral and caudal dorsal reticular nucleus

73

(DRt) and ventrolateral medulla (VLM)— play a role in determine the experience of pain. 15. A [16] All of the above are recognized causes of persistent spinal cord pain. Trauma is the predominant cause, and trauma due to motor vehicle accident is the largest subset, reporting 60–70% of traumas. 1 6. D [16, 17] While all of the entities listed above may cause central neuropathic pain, syringomyelia is most often associated with delayed onset of symptoms. Posttraumatic syringomyelia, the development of an expanding cyst in the center of the spinal cord, may develop slowly over years with insidious onset of symptoms. Time to development of syringomyelia is unrelated to the severity of spinal cord injury or of the initial trauma. The syrinx may extend above and/or below the level of the initial lesion. The most common initial symptom is reduced sensation in the region corresponding to the cord lesion (“suspended sensory level”). Dysesthetic pain, if it develops, usually follows later. Untreated syringomyelia may lead to marked neurologic deterioration. Surgical management is usually through syringoperitoneal shunt, but its benefits are not clear. 1 7. C [18] Acute, subacute, and chronic pain syndromes are described in multiple sclerosis (MS). Approximately 10% of patients have acute pain syndromes, and 50–80% have chronic pain. Trigeminal neuralgia is the most common cause of acute pain and may be the first manifestation of MS in younger patients, before the age of 50. It tends to occur at an earlier age and is more often bilateral. Chronic pain syndromes are almost always associated with a myelopathy. Examples of chronic pain are dysesthetic limb pain, back pain, painful leg spasms, and ­visceral pain from neurogenic bladder and bowel. Continuous burning lower limb pain occurs in onethird of the patients, and is common sequelae of spinothalamic sensory loss. 66–95% of

10  Central Pain

74

patients with dysesthetic pain manifest loss of temperature and sensation. Opioids are indicated to treat severe pain. In absence of history of substance abuse opioid treatment generally carries a low risk of psychologic dependence. Steroids are indicated to treat ocular pain from optic neuritis and painful leg spasms during disease relapse. Anticonvulsants are first-line agents for paroxysmal pain in MS. Carbamazepine is the drug of first choice in trigeminal neuralgia. Tricyclic antidepressants are mainstays of therapy for chronic dysesthetic pain. 18. C [19–22] Pain is a well-known and often overlooked consequence of stroke. Stroke patients can have nociceptive or neuropathic pain and the latter can be peripherally or centrally located. Central post-stroke pain (CPSP) is a specific type of centrally mediated neuropathic pain seen in a small proportion (8%) of patients after a stroke. It is characterized by occurrence of pain and somatosensory abnormalities (negative and/or positive symptoms and signs) in a stroke affected body region. Nociceptive pain, localized to the shoulder resulting from muscle tone and immobility following a stroke, is not CPSP. There is no evidence that CPSP has a psychogenic basis. 1 9. B [19–22] Despite the seminal report of ‘thalamic pain’ in association with stroke by Dejerine and Roussy, a lesion in the thalamus occurs in less than one-third (12–33%) of CPSP patients. Most stroke lesions (75%) are extrathalamic. It is generally agreed that damage to the spinothalamocortical pathway plays a significant role in the pathogenesis of CPSP. There is a wide range of time lag between the precipitating event and onset of CPSP symptoms. However, 40–60% develop CPSP within 1 month of stroke and a large majority within 6 months of stroke. A striking and universal feature of CPSP is disturbances in temperature sensibility marked by increase in pain on cold exposure, and abnormal sen-

sibility to non-noxious and/or noxious cold and warm stimuli. 2 0. A [15, 19, 21, 23–25] There is a general consensus that amitriptyline is the drug of first choice. It was the first tricyclic antidepressant to be tested in CPSP and was the first agent to demonstrate efficacy in a comparison trial with carbamazepine and in a placebo-controlled trial. Spinal cord stimulation has no role in CPSP. A small segment of CPSP patients has been found to respond well to long-term oral morphine. A single blind placebo-controlled study in central pain concluded that central pain is poorly responsive but not totally unresponsive to opioids. Lamotrigine monotherapy at 200 mg/day compared to placebo in 27 CPSP patients was found to be moderately effective. Side effects were mild. A major concern with lamotrigine therapy is the potential risk of Stevens Johnson syndrome. Motor cortex stimulation, first described in 1991, has shown to alleviate pain in CPSP.  Presently the technique is recommended in exceptional cases of CPSP where other treatments have failed.

References 1. Hong JH, Bai DS, Jeong JY, Choi BY, Chang CH, Kim SH, Ahn SH, Jang SH.  Injury of the spino-­thalamo-­ cortical pathway is necessary for central post-stroke pain. Eur Neurol. 2010;64(3):163–8. https://doi. org/10.1159/000319040. Epub 2010 Aug 10. PMID: 20699616. 2. Harrison RA, Field TS.  Post stroke pain: identification, assessment, and therapy. Cerebrovasc Dis. 2015;39(3–4):190–201. 3. Bendok B, Levy R.  Brain stimulation for persistent pain management. In: Gildenberg PL, Tasker R, editors. Textbook of stereotactic and functional neurosurgery. New York: McGraw-Hill; 1998. p. 1539–46. 4. Yamamoto T, Katayama Y, Hirayama T, Tsubokawa T. Pharmacological classification of central post-stroke pain: comparison with the results of chronic motor cortex stimulation therapy. Pain. 1997;72(1–2):5–12. 5. Bonicalzi V.  Central pain syndrome: elucidation of genesis and treatment. Expert Rev Neurother. 2007;7(11):1485–97.

References 6. Huang-Lionnet JH, Brummett C, Raja SN.  Central pain states. In: Benzon HT, editor. Essentials of pain medicine. Elsevier; 2018. 7. Ziconotide. 2020. https://pdr.net/drug-­summary/ Prialt-­ziconotide-­782.1142#11. 8. Blumenfeld H. Neuroanatomy through clinical cases. Sunderland, MA: Sinauer Associates; 2002. 9. Hanai F.  C fiber responses of wide dynamic range neurons in the spinal dorsal horn. Clin Orthop Relat Res. 1998;349:256–67. 10. Fields HL, Rowbotham M, Baron R.  Postherpetic neuralgia: irritable nociceptors and deafferentation. Neurobiol Dis. 1998;5(4):209–27. 11. Gierthmühlen J, Maier C, Baron R, et  al. Sensory signs in complex regional pain syndrome and peripheral nerve injury. Pain. 2012;153(4):765–74. 12. Haanpää M, Attal N, Backonja M, et  al. NeuPSIG guidelines on neuropathic pain assessment. Pain. 2011;152(1):14–27. 13. Hovaguimian A, Gibbons CH.  Diagnosis and treatment of pain in small-fiber neuropathy. Curr Pain Headache Rep. 2011;15(3):193–200. 14. Yarnitsky D.  Conditioned pain modulation (the diffuse noxious inhibitory control-like effect): its relevance for acute and chronic pain states. Curr Opin Anaesthesiol. 2010;23(5):611–5. 15. Bowsher D.  Central post-stroke (‘thalamic syn drome’) and other central pains. Am J Hosp Palliat Care. 1999;16:593–7. 16. Nicholson BD.  Evaluation and treatment of cen tral pain syndromes. Neurology. 2004;62(Suppl 2): S30–6.

75 17. Carroll AM, Brackenridge P.  Post-traumatic syrin gomyelia: a review of the cases presenting in a regional spinal injuries unit in the north east of England over a 5-year period. Spine. 2005;30(10): 1206–10. 18. Moulin DE. Pain in central and peripheral demyelinating disorders: multiple Sclerosis and Guillain-Barré syndrome. Neuropathic pain syndromes. Neurol Clin. 1998;16(4):889–97. 19. Hanson P. Post stroke pain case study: clinical characteristics, therapeutic options and long-term follow up. Eur J Neurol. 2004;11:22–30. 20. Andersen G, et  al. Incidence of central post-stroke pain. Pain. 1995;61:187–93. 21. Leijon G, Boivie J.  Central post stroke pain—neurological symptoms and pain characteristics. Pain. 1989;36:13–25. 22. Widar M, et  al. Long-term pain conditions after a stroke. J Rehabil Med. 2002;34:165–70. 23. Chen B, et al. Central post-stroke pain syndrome: yet another use for gabapentin? Am J Phys Med Rehabil. 2002;81:718–20. 24. Vestergaard K, et  al. Lamotrigine for central post stroke pain: a randomized controlled trial. Neurology. 2001;56:184–90. 25. Nguyen JP, et  al. Chronic motor cortex stimulation in the treatment of central and neuropathic pain. Correlations between clinical electrophysiological and anatomical data. Pain. 1999;82:245–51.

Headache Disorders

Questions 1. Which of the following mechanisms is not currently considered the etiology of migraine headaches A. Activation of the trigeminovascular system B. Cortical spreading depression C. Neuronal sensitization D. Vasoconstriction of cranial blood vessels 2. A 43-year-old woman presents for initial evaluation of headaches lasting approximately 12  h twice weekly. She describes unilateral headaches with a pulsating, throbbing quality with associated nausea and emesis. Her headaches are preceded by an aura which she describes as temporary vision loss followed by ringing in her ears. Which of the following medications could be considered for acute treatment? A. Sumatriptan B. Propranolol C. Phenothiazine D. Dihydroergotamine 3. An 18-year-old woman complains of unilateral headaches lasting 12–14  h if left untreated. She has no visual symptoms prior to the headache but frequently notices decreased strength in the right arm prior to onset of headache lasting 30  min. She has

11

also noted difficulty with speech during the headache. MRI of the head was read as normal. She would be classified as: A. Migraine with aura B. Hemiplegic migraine C. Basilar migraine D. Tension headache 4. A 34-year-old man with chronic migraines presents for follow up. Over the last several months, his headache frequency has increased to the point of using sumatriptan 1  day/week initially to currently 6  days/ week. He has missed at least 2  days per week of work over the last 4  weeks due to migraines. What is the next step in the management of this patient A. Start prophylactic treatment with pro pranolol B. Discontinue sumatriptan and provide short course of oral steroids C. Discontinue sumatriptan and provide anti-emetic D. Attempt sphenopalatine ganglion block 5. A 22-year-old female with migraines occurring twice weekly lasting 8–12 h each time is seen in the pain clinic. Her primary care provider has started her on sumatriptan for prophylaxis though the maximum recommended dose does not allow her to treat all of her headaches. Vitals include HR 45, BP

© The Author(s), under exclusive license to Springer Nature Switzerland AG 2021 M. Suer, N. Sehgal, Questions and Answers in Pain Medicine, https://doi.org/10.1007/978-3-030-68204-0_11

77

78

100/65, RR 14. Which of the following is the next step in management of this patient? A. Start on propranolol for prophylactic treatment B. Start on topiramate for prophylactic treatment C. Start on amitriptyline for prophylactic treatment D. Start on botulinum toxin injections for prophylactic management 6. A 33-year-old avid, albeit slow, triathlete suffers from occasional visual auras following difficult workouts described as “shimmering lights resulting in tunnel vision.” He recalls these happening since high school sports. He reports only having one migraine headache in his lifetime with pain, nausea, and photophobia. What is the next step in the management of this patient? A. Prescribe sumatriptan for migraine abortion B. Recommend use of NSAID and caffeine post workout C. No treatment is recommended D. Obtain MRI of the head 7. You decide to start an individual with chronic migraines on botulinum toxin for prophylactic treatment. Which of the following dosage-­location correlations is INCORRECT? A. Occipitalis: 30 units divided into 6 sites B. Frontalis: 20 units divided into 4 sites C. Procerus: 10 units divided into 2 sites D. Trapezius: 30 units divided into 6 sites 8. Which of the following is the MOST common side effect noted for botulinum toxin in the treatment of chronic migraine? A. Neck pain B. Headache C. Ptosis D. Facial paresis 9. A 28-year-old with a 5-year history of migraines treated with sumatriptan is seen in follow up. She and her husband are trying to get pregnant. Her migraine frequency has not changed from three migraines per month. What is the best management of this patient’s migraines going forward

11  Headache Disorders

A. Discontinue sumatriptan and start prophylactic treatment with propranolol B. Discontinue sumatriptan in favor of Isometheptane (Midrin) for abortive treatment C. Discontinue sumatriptan in favor of NSAIDs and caffeine for abortive treatment D. No changes are recommended in her migraine plan 10. A 33-year-old man has nightly, severe unilateral headaches lasting 15–20  min with associated eye redness and tearing. He had a similar presentation 1  year ago after which the headaches ceased after 2 months. Oxygen has provided some relief, but he is requesting a prophylactic medication. Which of the following is NOT considered a first-­ line prophylactic treatment for this type of headache? A. Propranolol 60 mg once daily B. Verapamil 240 mg daily C. Lithium 300 mg thrice day D. Methylprednisolone 40 mg once daily 11. Which of the following headache types occurs more commonly in women than men? 1. Cluster headaches 2. SUNCT 3. Paroxysmal Hemicrania 4. Migraine A. 1, 2, 3 B. 2, 3, 4 C. 3, 4 D. 1, 2 E. 4 12. A 25-year-old female presents with unre mitting, unilateral headaches exhibiting a frequency 10 per day. The headaches last 5–10  min each. The pain is described as throbbing localized over the right eye and temple. She frequently has unilateral tearing and eye redness with the headaches. MRI of her head reveals no abnormalities. What is the next step in the management of this patient? A. Administer indomethacin B. Administer oxygen via non-rebreather mask at 12 L/min

Questions

C. Administer sumatriptan D. Perform right supraorbital nerve block 13. A 72-year-old woman presents with new onset generalized aching pain in her neck, shoulders, upper back, and hip girdle. She also reports feeling fatigued over the last 6  weeks. She reports mild headaches over this time as well. Neurologic examination is normal. What is the next step in management of this patient A. MRI of the head B. Obtain ESR and CRP C. Start on pregabalin D. Start high dose steroids 14. You evaluate a 31-year-old woman for new onset headaches. She reports worse headaches in the morning with a pulsating whooshing sensation in the ears. You see the following on fundoscopic examination. What is the next step in management of this individual? A. Spinal tap to remove fluid and reduce pressure B. Recommend weight loss and reduced salt intake C. Referral to neurosurgery for consideration of shunt D. MRI of the head 15. A 28-year-old man presents for evaluation of chronic sinus headaches. He has been evaluated by his primary care and treated with many sinus treatments to no avail including nasal rinse, antibiotics, antihistamines, and OTC sinus medications. Intracranial imaging has failed to reveal an underlying sinus lesion. His examination is benign. Which of the following is the individual LEAST likely to be suffering from? A. Migraine B. Chronic tension type headaches C. Medication overuse headache D. Chronic bacterial infection. 16. Which of the following is NOT a risk factor for chronic daily headache A. History of migraine B. Obesity (BMI >30)

79

C. Low caffeine consumption D. Snoring 17. You are designing your new practice and wish to have all patients fill out questionnaires to track treatments. You wish to have a migraine-specific questionnaire to track patient’s headache-related disability with regards to work; household chores; and family, social, and leisure activities over a 3-month period. Which of the following questionnaires would you administer? A. Headache Impact Test (HIT) B. Headache Needs Assessment (HANA) Survey C. Henry Ford Hospital Disability Inventory (HDI) D. Migraine Disability Assessment (MIDAS) Questionnaire 18. Which of the following is FALSE regarding an aura A. Aura usually develops over 5–20  min and lasts less than 60 min B. An individual has one type of aura C. Can be visual, sensory, or motor D. Headache usually follows within 60 min of the end of the aura 19. A 22-year old female with chronic migraines is started on propranolol 80  mg/day for migraine prophylaxis. Two weeks after starting the medication, he reports having no ill effects of the medication but has not noted a decrease in her headache frequency. Vital signs: HR 82, BP 138/81, RR 16, O2 saturation 98% no room air. What is the next step in the management of this patient? A. Taper propranolol and start a second oral prophylactic medication B. Continue propranolol and add a second oral prophylactic medication C. Continue propranolol at an increased dosage D. Continue propranolol and perform botulinum toxin injections for prophylaxis 20. A 22-year old female with chronic head aches was recently started on an herbal medication for headache treatment. She noted no

11  Headache Disorders

80

improvement and discontinued it. She has developed tachycardia following cessation. Which of the following herbal medications was she likely started on A. St John’s Wort B. Feverfew C. Ginseng D. Ashwagandha 21. Which of the following is a characteristic of tension-type headache? A. Pulsating quality B. Bilateral pain C. Nausea or vomiting D. Aggravation by walking stairs or similar routine physical activity E. Photophobia and phonophobia 22. A 40-year-old man presents with a 1-week history of unilateral, excruciating episodic headaches accompanied by ipsilateral lacrimation. Each episode lasts for about 15 min, and occurs two or three times/day. It wakes him up from sleep. MRI scan of the brain is normal. Examination reveals ptosis and lacrimation on the side ipsilateral to the headache, present only when the headache is occurring. You would expect to observe the following: 1. Benefit from inhalation of 100% oxygen at start of headache attack 2. Ipsilateral hyperreflexia 3. Benefit from injection of sumatriptan at start of headache attack 4. Remission with chronic opioid therapy A. 1, 2 and 3 B. 1 and 3 C. 2 and 4 D. 4 only 23. A 49-year-old woman presents to the primary care clinic with a 2-month history of worsening headache, starting at 1/10 intensity but now at 8/10. She has never experienced any severe headache before, only occasional mild headaches. The headache has awakened her in the middle of the night. Position changes seem to make it worse. She has vomited on three occasions in the last week, but has no prior history of GI problems or recent gastrointestinal infection. Your next moves should include:

1 . A thorough neurological examination 2. An urgent cranial CT scan 3. Triptan therapy 4. Lumbar puncture A. 1, 2 and 3 B. 1 and 2 C. 2 and 4 D. 2 only

Answers 1. D [1] Classically, migraines are described as initiated by vasodilation whereas headaches are due to vasoconstriction. During a migraine, inflammation of the tissue surrounding the brain, i.e., neurogenic inflammation, exacerbates the pain. Therefore, medicine often prescribed to treat a headache, such as beta-blockers, dilate the blood vessels and therefore can make a migraine worse. Hence, many of the medications used to treat migraines are vasoconstrictors. 2. C [2, 3] Ergotamine derivatives, beta blockers, and triptans should be avoided in patients with basilar migraines because of theoretical concerns and anecdotal evidence that these medications may lead to cerebral ischemia. A typical aura in basilar migraine might present with bilateral visual loss or blindness. Following, or independent of, the visual phenomena, patients may complain of vertigo, dysarthria, diplopia, tinnitus, ataxia, a decreased level of consciousness, or bilateral sensory (paresthesias) or subjective motor symptoms (there should be no objective weakness); sometimes nausea and emesis are prominent. 3. B [4] Hemiplegic migraine is described as the following: • At least two attacks of migraine with a reversible aura of motor weakness that can last 1 h to days • Also includes one of the following:

Answers

–– Positive or negative visual symptoms –– Positive or negative sensory symptoms –– Dysphasia or dysarthria Frequently accompanied by symptoms typical of basilar migraine • If at least one first- or second-degree relative has a migrainous aura that includes motor weakness, it is familial hemiplegic migraine and is associated with a mutation in the neuronal calcium channel. • If no first- or second-degree relative has a migrainous aura that includes motor weakness, it is sporadic hemiplegic migraine. 4. C [5] This individual is suffering from medication overuse or rebound headaches. Chronic use of migraine treatments—triptans, NSAIDs, acetaminophen, butalbital, narcotics, ergotamine, DHE, and isometheptane—more than twice per week should be discouraged. Prophylactic regimens generally are not effective in the setting of rebound. The treatment of medication overuse headache is discontinuation of all analgesics. The physiologic washout period, during which patients may continue to experience frequent headaches, lasts at least 2 weeks; patients should continue to refrain from analgesic medications for a total of 10–12 weeks. The initial several days to 2 weeks following analgesic withdrawal might be the most difficult and are frequently punctuated with a severe rebound headache; antiemetics and maintaining hydration, as well as patience, are effective. Using steroids during this period has no effect on outcome. 5. B [4] A. Propranolol is not the best first choice in this individual given the relatively low resting heart rate and blood pressure. B. Correct. This is likely the best answer given the potential side effects noted with the other options. C. Antidepressants are relatively contraindicated in patients who use triptans, as they may suffer from excessive sero-

81

tonin stimulation (serotonin syndrome), although the incidence of this complication is virtually negligible unless the patient is on very high doses of the antidepressant. D. While often used as prophylactic treatment quite successfully, the first step in management is oral medications prior to initiating botulinum toxin injections. While these can be quite successful, one should remember the costs associated keeping in mind most insurance companies would request oral medication trial prior to initiating botulinum toxin. 6. B [6, 7] This answer choice is less clear than many others. While this individual does not have headaches, the visual aura can be quite disturbing and unsettling for many individuals. Frequently, when occurring post-workout, individuals can benefit from caffeine and NSAIDs although one must ensure the safety of these medications in each patient. Certainly sumatriptan (A) would not be the proper choice given the lack of migraine headaches. MRI of the head is not indicated in this individual with no recent changes in his symptoms. One could argue for no treatment given the lack of headaches, but if this individual is interested enough in treatment to present to the physician, one could assume the symptoms are bothersome enough to warrant fairly conservative treatments. 7. C [8] The recommended dilution is 200  Units/4  mL or 100  Units/2  mL, with a final concentration of 5  Units per 0.1 mL. The recommended dose for treating chronic migraine is 155  Units administered intramuscularly divided as: • Corrugator: 5 units each side • Procerus: 1 site (midline) • Frontalis: 10 units each side • Temporalis: 20 units each side • Occipitalis: 15 units each side • Cervical paraspinals: 10 units each side • Trapezius: 15 units each side

11  Headache Disorders

82

8. A [9] The most frequently reported adverse reactions following injection of botulinum for Chronic Migraine vs placebo include • Neck pain (9% vs 3%) • Headache (5% vs 3%) • Eyelid ptosis (4% vs 20 mg/dL. An unusually hot or cold urine, small sample volume (